Chapter 10

You might also like

Download as pdf or txt
Download as pdf or txt
You are on page 1of 113

© 2016 Pearson Education, Inc., Upper Saddle River, NJ. All rights reserved.

This material is protected under all copyright laws as they currently


exist. No portion of this material may be reproduced, in any form or by any means, without permission in writing from the publisher.

10–1.

Determine the moment of inertia about the x axis. y

y  bn xn
a
b

x
a

Solution
a 1
Differential Element. Here x = y n . The area of the differential element parallel
1
b n
a 1
to the x axis shown shaded in Fig. a is dA = (a - x)dy = aa - 1 y n bdy.
bn
Moment of Inertia. Perform the integration,
b

LA L0
a 1
Ix = y2dA = y2aa - 1
yn bdy
b
n

L0
a 1
= aay2 - 1
yn + 2 bdy
b
n

b
a a n 3n + 1
= c y3 - a 1 ba by d3
3 bn 3n + 1 n
0

1 3 n
= ab - a bab3
3 3n + 1

ab3
= Ans.
3(3n + 1)

Ans:
ab3
Ix =
3(3n + 1)

1010
© 2016 Pearson Education, Inc., Upper Saddle River, NJ. All rights reserved. This material is protected under all copyright laws as they currently
exist. No portion of this material may be reproduced, in any form or by any means, without permission in writing from the publisher.

10–2.

Determine the moment of inertia about the y axis. y

y  bn xn
a
b

x
a

Solution
Differential Element. The area of the differential element parallel to the y axis
b
shown shaded in Fig. a is dA = ydx = n xndx.
a
Moment of Inertia. Perform the integration,
a

LA L0
b n
Iy = x2dA = x2 a x dxb
an
a

L0 a
b n+2
= nx dx

a
b 1
= na b(xn + 3) `
a n + 3 0

a3b
= Ans.
n + 3

Ans:
a3b
Iy =
n + 3

1011
© 2016 Pearson Education, Inc., Upper Saddle River, NJ. All rights reserved. This material is protected under all copyright laws as they currently
exist. No portion of this material may be reproduced, in any form or by any means, without permission in writing from the publisher.

10–3.

Determine the moment of inertia for the shaded area about y


the x axis.
100 mm

200 mm y  1 x2
50

Solution
1
Differential Element. Here x = 250y2 . The area of the differential element parallel x
1
to the x axis shown shaded in Fig. a is dA = 2x dy = 2250y2dy.
Moment of Inertia. Perform the integration,
200 mm

LA L0
1
Ix = y2dA = y2 c 2250y2dy d

200 mm

L0
5
= 2250 y2dy

200 mm
2 7
= 2250 a y2 b 3
7
0

= 457.14(106) mm4

= 457(106) mm4  Ans.

Ans:
Ix = 457 ( 106 ) mm4

1012
© 2016 Pearson Education, Inc., Upper Saddle River, NJ. All rights reserved. This material is protected under all copyright laws as they currently
exist. No portion of this material may be reproduced, in any form or by any means, without permission in writing from the publisher.

*10–4.

Determine the moment of inertia for the shaded area about y


the y axis.
100 mm

200 mm y  1 x2
50

Solution
1
Differential Element. Here x = 250 y2 . The moment of inertia of the differential x
element parallel to x axis shown in Fig. a about y axis is
1 2 2 1 100250 3
dIy = (dy)(2x)3 = x3dy = ( 250y2 ) 3dy = y2dy.
12 3 3 3
Moment of Inertia. Perform the integration,
200 mm

L L0
100150 3
Iy = dIy = y2dy
3
200 mm
100250 2 5 3
= a y2 b
3 5
0

= 53.33(106) mm4
= 53.3(106) mm4  Ans.

Ans:
Iy = 53.3(106) mm4

1013
© 2016 Pearson Education, Inc., Upper Saddle River, NJ. All rights reserved. This material is protected under all copyright laws as they currently
exist. No portion of this material may be reproduced, in any form or by any means, without permission in writing from the publisher.

10–5.

Determine the moment of inertia for the shaded area about y


the x axis.
y  x1/ 2

1m

x
1m

Solution
Differential Element. The area of the differential element parallel to the y axis
shown shaded in Fig. a is dA = ydx. The moment of inertia of this element about
the x axis is
dIx = dIx′ + dA ∼
y2
1 y 2
= (dx)y3 + ydx a b
12 2
1 3
= y dx
3
1 1 3
= (x2) dx
3
1 3
=x2 dx
3
Moment of Inertia. Perform the integration.
1m

L L0
1 3
Ix = dIx = x2dx
3
1m
2 53
= x2
15
0

2 4
= m = 0.133 m4 Ans.
15

Ans:
Ix = 0.133 m4

1014
© 2016 Pearson Education, Inc., Upper Saddle River, NJ. All rights reserved. This material is protected under all copyright laws as they currently
exist. No portion of this material may be reproduced, in any form or by any means, without permission in writing from the publisher.

10–6.

Determine the moment of inertia for the shaded area about y


the y axis.
y  x1/ 2

1m

x
1m

Solution
Differential Element. The area of the 1differential element parallel to the y axis
shown shaded in Fig. a is dA = ydx = x2dx.
Moment of Inertia. Perform the integration,
1m

LA L0
1
Iy = x2dA = x2 ( x2dx )

1m
2 7
= x2 3
7
0

2 4
= m = 0.286 m4 Ans.
7

Ans:
Iy = 0.286 m4

1015
© 2016 Pearson Education, Inc., Upper Saddle River, NJ. All rights reserved. This material is protected under all copyright laws as they currently
exist. No portion of this material may be reproduced, in any form or by any means, without permission in writing from the publisher.

10–7.

Determine the moment of inertia for the shaded area about y


the x axis.
y2  1  0.5x

1m

x
2m

Solution
Differential Element. Here x = 2(1 - y2). The area of the differential element
parallel to the x axis shown shaded in Fig. a is dA = xdy = 2(1 - y2)dy.

Moment of Inertia. Perform the integration,


1m

LA L0
Ix = y2dA = y2[2(1 - y2)dy]

1m

L0
= 2 (y2 - y4)dy

1m
y3 y5
= 2a - b 3
3 5
0
4 4
= m = 0.267 m4  Ans.
15

Ans:
Ix = 0.267 m4

1016
© 2016 Pearson Education, Inc., Upper Saddle River, NJ. All rights reserved. This material is protected under all copyright laws as they currently
exist. No portion of this material may be reproduced, in any form or by any means, without permission in writing from the publisher.

*10–8.

Determine the moment of inertia for the shaded area about y


the y axis.
y2  1  0.5x

1m

x
2m

Solution
Differential Element. Here x = 2(1 - y2). The moment of inertia of the differential
element parallel to the x axis shown shaded in Fig. a about the y axis is
∼2
dIy = dIy' + dAx
1 x 2
= (dy)x3 + xdya b
12 2
1 3
= x dy
3
1
= 32(1 - y2) 4 3 dy
3
8
=( - y6 + 3y4 - 3y2 + 1)dy
3
Moment of Inertia. Perform the integration,
1m

L 3 L0
8
Iy = dIy = ( -y6 + 3y4 - 3y2 + 1)dy

8 y7 3 1m
= a - + y5 - y3 + yb `
3 7 5 0

128 4
= m = 1.22 m4 Ans.
105

Ans:
Iy = 1.22 m4

1017
© 2016 Pearson Education, Inc., Upper Saddle River, NJ. All rights reserved. This material is protected under all copyright laws as they currently
exist. No portion of this material may be reproduced, in any form or by any means, without permission in writing from the publisher.

10–9.

Determine the moment of inertia of the area about the x y


axis. Solve the problem in two ways, using rectangular
differential elements: (a) having a thickness dx and y = 2.5 – 0.1x2
(b) having a thickness of dy.
2.5 ft
x
5 ft

SOLUTION
(a) Differential Element: The area of the differential element parallel to y axis is
dA = ydx. The moment of inertia of this element about x axis is
'
dIx = dIx¿ + dAy 2

1 y 2
= 1dx2y 3 + ydxa b
12 2

1
= 12.5 - 0.1x223 dx
3

1
= 1 - 0.001x6 + 0.075x4 - 1.875x2 + 15.6252 dx
3

Moment of Inertia: Performing the integration, we have


5 ft
1
Ix = dIx = 1 -0.001x6 + 0.075x4 - 1.875x2 + 15.6252 dx
L 3 L-5 ft

x + 15.625xb `
5 ft
1 0.001 7 0.075 5 1.875 3
= a- x + x -
3 7 5 3 -5 ft

= 23.8 ft4 Ans.

(b) Differential Element: Here, x = 225 - 10y. The area of the differential
element parallel to x axis is dA = 2xdy = 2 225 - 10y dy.

Moment of Inertia: Applying Eq. 10–1 and performing the integration, we have

Ix = y2dA
LA
2.5 ft
= 2 y2 225 - 10ydy
L0
y2
125 - 10y22 d `
2y 2.5 ft
2
= 2c - 125 - 10y22 - 125 - 10y22 -
3 3 7

15 375 13125 0

= 23.8 ft4 Ans.

Ans:
Ix = 23.8 ft 4

1018
© 2016 Pearson Education, Inc., Upper Saddle River, NJ. All rights reserved. This material is protected under all copyright laws as they currently
exist. No portion of this material may be reproduced, in any form or by any means, without permission in writing from the publisher.

10–10.

Determine the moment of inertia for the shaded area about y


the x axis.
y2 h2
—x
b

x
b
SOLUTION
1 3
d Ix = y dx
3

Ix = d Ix
L
b 3 b
y 1 h2 3>2 3>2
= dx = a b x dx
L0 3 L0 3 b

1 h2 3>2 2 5>2 b
= a b a b x ]0
3 b 5

2
= bh3 Ans.
15

Also,

b 2
dA = (b - x) dy = (b - y ) dy
h2

Ix = y2 dA
L
h
b 2
= y2 (b - y ) dy
L0 h2

b b 5 h
= c y3 - y d
3 5h2 0

2
= bh3 Ans.
15

Ans:
2
Ix = bh3
15

1019
© 2016 Pearson Education, Inc., Upper Saddle River, NJ. All rights reserved. This material is protected under all copyright laws as they currently
exist. No portion of this material may be reproduced, in any form or by any means, without permission in writing from the publisher.

10–11.

Determine the moment of inertia for the shaded area about y


the x axis.

8m

y  1 x3
8
x
Solution 4m
1
Differential Element. Here, x = 2y3. The area of the1 differential element parallel to
the x axis shown shaded in Fig. a is dA = xdy = 2y3 dy

Moment of Inertia. Perform the integration,


8m

LA L0
1
2 2
Ix = y dA = y (2y3 dy)

8m

L0
7
= 2 y3 dy

3 10 8 m
= 2a y3 b `
10 0

= 614.4 m4 = 614 m4 Ans.

Ans:
Ix = 614 m4

1020
© 2016 Pearson Education, Inc., Upper Saddle River, NJ. All rights reserved. This material is protected under all copyright laws as they currently
exist. No portion of this material may be reproduced, in any form or by any means, without permission in writing from the publisher.

*10–12.

Determine the moment of inertia for the shaded area about y


the y axis.

8m

y  1 x3
8
x
Solution 4m

Differential Element. The area of the differential element parallel to the y axis,
1
shown shaded in Fig. a, is dA = (8 - y)d x = a8 - x3 bdx
8
Moment of Inertia. Perform the integration,

4m

LA L0
1 3
Iy = x2dA = x2 a8 - x b dx
8
4m

L0
1 5
= a8x2 - x b dx
8
4m
8 1 6 3
= a x3 - x b
3 48
0

= 85.33 m4 = 85.3 m4 Ans.

Ans:
Iy = 85.3 m4

1021
© 2016 Pearson Education, Inc., Upper Saddle River, NJ. All rights reserved. This material is protected under all copyright laws as they currently
exist. No portion of this material may be reproduced, in any form or by any means, without permission in writing from the publisher.

10–13.

Determine the moment of inertia about the x axis. y

x2  4y2  4

1m

x
2m

Solution
1
Differential Element. Here, y = 24 - x2. The moment of inertia of the differential
2
element parallel to the y axis shown shaded in Fig. a about x axis is

∼2
dIx = dIx′ + dAy
1 y 2
= (dx)y3 + ydx a b
12 2

1 3
= y dx
3
3
1 1
= a 24 - x2 b dx
3 2

1
= 2(4 - x2)3 dx
24
Moment of Inertia. Perform the integration.
2m

L L0
1
Ix = dIx = 2(4 - x2)3 dx
24

1 x 2m
= c x2(4 - x2)3 + 6x24 - x2 + 24 sin - 1 d `
96 2 0

p 4
= m Ans.
8

Ans:
p 4
Ix = m
8

1022
© 2016 Pearson Education, Inc., Upper Saddle River, NJ. All rights reserved. This material is protected under all copyright laws as they currently
exist. No portion of this material may be reproduced, in any form or by any means, without permission in writing from the publisher.

10–14.

Determine the moment of inertia about the y axis. y

x2  4y2  4

1m

x
2m

Solution
1
Differential Element. Here, y = 24 - x2. The area of the differential element
2
1
parallel to the y axis shown shaded in Fig. a is dA = ydx = 24 - x2dx
2

Moment of Inertia. Perform the integration,


2m

LA L0
1
Iy = x2dA = x2 c 24 - x2dx d
2
2m

2 L0
1
= x2 24 - x2dx

2m
1 x 1 x
= c - 2(4 - x2)3 + ax24 - x2 + 4 sin - 1 b d `
2 4 2 2 0

p 4
= m  Ans.
2

Ans:
p 4
Iy = m
2

1023
© 2016 Pearson Education, Inc., Upper Saddle River, NJ. All rights reserved. This material is protected under all copyright laws as they currently
exist. No portion of this material may be reproduced, in any form or by any means, without permission in writing from the publisher.

10–15.

Determine the moment of inertia for the shaded area about y


the x axis.

4 in.
y2  x
x
16 in.

Solution
Differential Element. The area of the differential element parallel with the x axis
shown shaded in Fig. a is dA = x dy = y2 dy.

Moment of Inertia. Perform the integration,


4 in.

LA L0
Ix = y2dA = y2(y2dy)

4 in.

L0
= y4dy

y5 4 in.
= `
5 0

= 204.8 in4 = 205 in4 Ans.

Ans:
Ix = 205 in4

1024
© 2016 Pearson Education, Inc., Upper Saddle River, NJ. All rights reserved. This material is protected under all copyright laws as they currently
exist. No portion of this material may be reproduced, in any form or by any means, without permission in writing from the publisher.

*10–16.

Determine the moment of inertia for the shaded area about y


the y axis.

4 in.
y2  x
x
16 in.

Solution
Differential Element. The moment of inertia of the differential element parallel to
the x axis shown shaded in Fig. a about the y axis is

∼2
dIy = dIy + dAx
1 x 2
= (dy)x3 + (xdy) a b
12 2
1 3
= x dy
3
1 23
= (y ) dy
3
1 6
= y dy
3

Moment of Inertia. Perform the integration,


4 in.

L L0
1 6
Iy = dIy = y dy
3

1 y7 4 in.
= a b`
3 7 0

  = 780.19 in4 = 780 in4 Ans.

Ans:
Iy = 780 in4

1025
© 2016 Pearson Education, Inc., Upper Saddle River, NJ. All rights reserved. This material is protected under all copyright laws as they currently
exist. No portion of this material may be reproduced, in any form or by any means, without permission in writing from the publisher.

10–17.

Determine the moment of inertia for the shaded area about y


the x axis.

h
y  h x3
b3

x
b

Solution
Differential Element. The moment of inertia of the differential element parallel to
the y axis shown shaded in Fig. a about the x axis is

∼2
dIx = dIx′ + dAy
1 y 2
= (dx)y3 + ydx a b
12 2
1 3
= y dx
3
1 h 3 3
= a x b dx
3 b3
h3 9
= x dx
3b9
Moment of Inertia. Perform the integration,
b
h3 9
L L0 3b
Ix = dIx = 9
x dx

h3 x10 b
= a b`
3b9 10 0
1 3
= bh  Ans.
30

Ans:
1
Ix = bh3
30

1026
© 2016 Pearson Education, Inc., Upper Saddle River, NJ. All rights reserved. This material is protected under all copyright laws as they currently
exist. No portion of this material may be reproduced, in any form or by any means, without permission in writing from the publisher.

10–18.

Determine the moment of inertia for the shaded area about y


the y axis.

h
y  h x3
b3

x
b

Solution
Differential Element. The area of the differential element parallel to the y axis
h
shown shaded in Fig. a is dA = ydx = 3 x3dx
b
Moment of Inertia. Perform the integration,
b

LA L0
h 3
Iy = x2dA = x2 a x bdx
b3
b

b3 L0
h
= x5dx

h x6 6
= a b`
b3 6 0
b3h
=  Ans.
6

Ans:
b3h
Iy =
6

1027
© 2016 Pearson Education, Inc., Upper Saddle River, NJ. All rights reserved. This material is protected under all copyright laws as they currently
exist. No portion of this material may be reproduced, in any form or by any means, without permission in writing from the publisher.

10–19.

Determine the moment of inertia for the shaded area about y


the x axis.
y2  1  x
1m

x
1m

1m

Solution
1
Differential Element. Here y = (1 - x)2 . The moment of inertia of the
differential element parallel to the y axis shown shaded in Fig. a about the x axis is
1 2 2 1 2 3
dIx = (dx)(2y)3 = y3dx = 3(1 - x)2 4 3 dx = (1 - x)2 dx.
12 3 3 3
Moment of Inertia. Perform the integration,
1m

L L0
2 3
Ix = dIx = (1 - x)2 dx
3
1m
2 2 5
= c - (1 - x)2 d `
3 5 0

4 4
= m = 0.267 m4 Ans.
15

Ans:
Ix = 0.267 m4

1028
© 2016 Pearson Education, Inc., Upper Saddle River, NJ. All rights reserved. This material is protected under all copyright laws as they currently
exist. No portion of this material may be reproduced, in any form or by any means, without permission in writing from the publisher.

*10–20.

Determine the moment of inertia for the shaded area about y


the y axis.
y2  1  x
1m

x
1m

1m

Solution
Differential Element. Here x = 1 - y2. The moment of inertia of the differential
element parallel to the x axis shown shaded in Fig. a about the y axis is
∼2
dIy = dIy′ + dAx

1 x 2
= (dy)x3 + xdy a b
12 2
1 3
= x dy
3
1
= (1 - y2)3dy
3
1
( -y6 + 3y4 - 3y2 + 1)dy
=
3
Moment of Inertia. Perform the integration,
1m

L L-1 m 3
1
Iy = dIy = ( - y6 + 3y4 - 3y2 + 1)dy

1 y7 3 1m
= ( - + y5 - y3 + y) `
3 7 5 -1 m

32 4
= m = 0.305 m4 Ans.
105

Ans:
Iy = 0.305 m4

1029
© 2016 Pearson Education, Inc., Upper Saddle River, NJ. All rights reserved. This material is protected under all copyright laws as they currently
exist. No portion of this material may be reproduced, in any form or by any means, without permission in writing from the publisher.

10–21.

Determine the moment of inertia for the shaded area about y


the x axis.
y2  2x

2m
yx

x
2m

Solution
1 2
Differential Element. Here x2 = y and x1 = y . The area of the differential element
2
1
parallel to the x axis shown shaded in Fig. a is dA = (x2 - x1)dy = ay - y2 bdy.
2
Moment of Inertia. Perform the integration,
2m

LA L0
1 2
Ix = y2dA = y2 ay - y bdy
2
2m

L0
1 4
= ay3 - y bdy
2
y4 y5 2 m
= a - b2
4 10 0
= 0.8 m4 Ans.

Ans:
Ix = 0.8 m4

1030
© 2016 Pearson Education, Inc., Upper Saddle River, NJ. All rights reserved. This material is protected under all copyright laws as they currently
exist. No portion of this material may be reproduced, in any form or by any means, without permission in writing from the publisher.

10–22.

Determine the moment of inertia for the shaded area about y


the y axis.
y2  2x

2m
yx

x
2m

Solution
1
Differential Element. Here, y2 = 22x2 and y1 = x. The area of the differential
element parallel to the y axis shown shaded in Fig. a is dA = (y2 - y1) dx
= 1 22x2 - x 2 dx.
1

Moment of Inertia. Perform the integration,


2m

LA L0
1
Iy = x2dA = x2 a 22x2 - xbdx

2m

L0
= 1 22x 5
2 - x3 2 dx

222 7 x4 2 2 m
= a x2 - b
7 4 0
4 4
= m = 0.571 m4 Ans.
7

Ans:
Iy = 0.571 m4

1031
© 2016 Pearson Education, Inc., Upper Saddle River, NJ. All rights reserved. This material is protected under all copyright laws as they currently
exist. No portion of this material may be reproduced, in any form or by any means, without permission in writing from the publisher.

10–23.

Determine the moment of inertia for the shaded area about y


the x axis. b2
y2  —x
a

b
b x2
y —
a2
x
a

Solution
a 1 a
Differential Element. Here x2 = 1
y2 and x1 = 2 y2. Thus, the area of the
b2 b
differential element parallel to the x axis shown shaded in Fig. a is dA = (x2 - x1) dy
a 1 a
= a 1 y2 - 2 y2 b dy.
b2 b
Moment of Inertia. Perform the integration,
b

LA L0
a 1 a
Ix = y2dA = y2 a 1 y2 - 2 y2 bdy
b2 b
b

L0
a 5 a
= a 1 y2 - 2 y4 bdy
b2 b
2a 7 a 5 2b
= a y2 - y b
5b2
1
7b 2 0

3ab3
= Ans.
35

Ans:
3ab3
Ix =
35

1032
© 2016 Pearson Education, Inc., Upper Saddle River, NJ. All rights reserved. This material is protected under all copyright laws as they currently
exist. No portion of this material may be reproduced, in any form or by any means, without permission in writing from the publisher.

*10–24.

Determine the moment of inertia for the shaded area about y


the y axis.
b2
y2  —x
a

b
b x2
y —
a2
x
a

Solution
b 1 b 2
Differential Element. Here, y2 = x2 and y1 =
x . Thus, the area of the
a2
1
a 2

differential element parallel to the y axis shown shaded in Fig. a is dA = (y2 - y1)dx
b 1 b
= a 1 x2 - 2 x2 b dx
a2 a
Moment of Inertia. Perform the integration,
a

LA L0
b 1 b
Iy = x2dA = x2 a 1 x2 - 2 x2 bdx
a2 a
a

L0 a
b 5 b
= a 1 x2 - 2 x4 bdx
2 a
2b 7 b 5 2a
= a x2 - x b
5a2
1
7a 2 0

2 3 1
= a b - a3b
7 5

3a3b
=  Ans.
35

Ans:
3a3b
Iy =
35

1033
© 2016 Pearson Education, Inc., Upper Saddle River, NJ. All rights reserved. This material is protected under all copyright laws as they currently
exist. No portion of this material may be reproduced, in any form or by any means, without permission in writing from the publisher.

10–25.

Determine the moment of inertia of the composite area y


about the x axis.
3 in. 6 in.

SOLUTION 3 in.

Composite Parts: The composite area can be subdivided into three segments as
shown in Fig. a. The perpendicular distance measured from the centroid of each 3 in.
segment to the x axis is also indicated.
x
Moment of Inertia: The moment of inertia of each segment about the x axis can be
determined using the parallel-axis theorem. Thus,
-
Ix = Ix¿ + A(dy)2

= B (3)(33) + (3)(3)(4)2 R + B (3)(33) + 3(3)(1.5)2 R


1 1 1
36 2 12

+ B (6)(63) + (6)(6)(2)2 R
1 1
36 2
= 209 in4 Ans.

Ans:
Ix = 209 in4

1034
© 2016 Pearson Education, Inc., Upper Saddle River, NJ. All rights reserved. This material is protected under all copyright laws as they currently
exist. No portion of this material may be reproduced, in any form or by any means, without permission in writing from the publisher.

10–26.

Determine the moment of inertia of the composite area y


about the y axis.
3 in. 6 in.

3 in.
SOLUTION
Composite Parts: The composite area can be subdivided into three segments as
shown in Fig. a. The perpendicular distance measured from the centroid of each 3 in.
segment to the x axis is also indicated.
x

Moment of Inertia: The moment of inertia of each segment about the y axis can be
determined using the parallel-axis theorem. Thus,
-
Iy = Iy ¿ + A(dx)2

= B (3)(33) + (3)(3)(2)2 R + B (3)(33) + 3(3)(1.5)2 R


1 1 1
36 2 12

+ B (6)(63) + (6)(6)(5)2 R
1 1
36 2
= 533 in4 Ans.

Ans:
Iy = 533 in4

1035
© 2016 Pearson Education, Inc., Upper Saddle River, NJ. All rights reserved. This material is protected under all copyright laws as they currently
exist. No portion of this material may be reproduced, in any form or by any means, without permission in writing from the publisher.

10–27.

The polar moment of inertia for the area is y¿


JC = 642 (106) mm4, about the z′ axis passing through the
centroid C. The moment of inertia about the y′ axis is
264 (106) mm4, and the moment of inertia about the x axis is
938 (106) mm4. Determine the area A.

C
x¿

200 mm
Solution
Applying the parallel-axis theorem with d y = 200 mm and Ix = 938 ( 106 ) mm4, x

Ix = Ix′ + Ad 2y

938 ( 106 ) = Ix′ + A ( 2002 )


Ix′ = 938 ( 106 ) - 40 ( 103 ) A


with known polar moment of inertia about C,

JC = Ix′ + Iy′

642 ( 106
) = 938 ( 106 ) - 40 ( 103 ) A + 264 ( 106 )
A = 14.0 ( 103 ) mm2
  Ans.

Ans:
A = 14.0 ( 103 ) mm2

1036
© 2016 Pearson Education, Inc., Upper Saddle River, NJ. All rights reserved. This material is protected under all copyright laws as they currently
exist. No portion of this material may be reproduced, in any form or by any means, without permission in writing from the publisher.

*10–28.

Determine the location y of the centroid of the channel’s 50 mm 50 mm


cross-sectional area and then calculate the moment of
inertia of the area about this axis.

x
250 mm
–y

SOLUTION
50 mm
Centroid: The area of each segment and its respective centroid are tabulated below.
350 mm
' '
Segment A (mm2) y (mm) y A (mm3)
1 100(250) 125 3.12511062

2 250(50) 25 0.312511062

© 37.511032 3.437511062

Thus,
' 3.437511062
' ©yA
y = = = 91.67 mm = 91.7 mm Ans.
©A 37.511032

Moment of Inertia: The moment of inertia about the x¿ axis for each segment can be
determined using the parallel-axis theorem Ix¿ = Ix¿ + Ad2y.

Segment Ai (mm2) (dy)i (mm) (Ix¿)i (mm4) (Ad 2y)i (mm4) (Ix¿)i (mm4)
1 3
1 100(250) 33.33 12 110021250 2 27.77811062 157.9911062
1 3
2 250(50) 66.67 12 12502150 2 55.55611062 58.1611062

Thus,

Ix¿ = © Ix¿ i = 216.15 106 mm4 = 216 106 mm4 Ans.

Ans:

y = 91.7 mm
Ix′ = 216(106) mm4

1037
© 2016 Pearson Education, Inc., Upper Saddle River, NJ. All rights reserved. This material is protected under all copyright laws as they currently
exist. No portion of this material may be reproduced, in any form or by any means, without permission in writing from the publisher.

10–29.

Determine y, which locates the centroidal axis x′ for the y¿


cross-sectional area of the T-beam, and then find the
moments of inertia Ix′ and Iy′.

75 mm
75 mm

y
20 mm

C
x¿
Solution
150 mm
Centroid. Referring to Fig. a, the areas of the segments and their respective centroids
are tabulated below.

∼ ∼A(mm3)
Segment A(mm2) y (mm) y
1 150(20) 10 30(103) 20 mm

2 20(150) 95 285(103)
Σ 6(103) 315(103)

∼2
Σy A 315(103)
Thus, y = = = 52.5 mm Ans.
ΣA 6(103)

Moment of Inertia. The moment of inertia about the x′ axis for each segment can
be determined using the parallel axis theorem, Ix′ = Ix′ + Ad 2y. Referring to Fig. b,

Segment Ai(mm2) (dy)i (mm) (Ix′)i (mm4) (Ad 2y)i (mm4) (Ix′)i (mm4)
1 150(20) 42.5 1 6
5.51875(106)
(150)(203) 5.41875(10 )
12
2 20(150) 42.5 1 6
11.04375(106)
(20)(1503) 5.41875(10 )
12
Thus

    Ix′ = Σ(Ix′)i = 16.5625(106) mm4 = 16.6(106) mm4 Ans.


Since the y' axis passes through the centroids of segments 1 and 2,
1 1
Iy′ = (20)(1503) + (150)(203)
12 12
= 5.725(106) mm4 Ans.

Ans:
y = 52.5 mm
Ix′ = 16.6 ( 106 ) mm4
Iy′ = 5.725 ( 106 ) mm4

1038
© 2016 Pearson Education, Inc., Upper Saddle River, NJ. All rights reserved. This material is protected under all copyright laws as they currently
exist. No portion of this material may be reproduced, in any form or by any means, without permission in writing from the publisher.

10–30.

Determine the moment of inertia for the beam’s cross- y


1 in.
sectional area about the x axis.

1 in.
8 in.

3 in.
x
1 in. 10 in.

Solution
Moment of Inertia. The moment of inertia about x axis for each segment can be
determined using the parallel axis theorem, Ix = Ix′ + Ad 2y. Referring to Fig. a,

Segment Ai(in2) (dy)i (in.) (Ix′)i (in4) (Ad 2y)i (in4) (Ix)i (in4)
1
1 1(8) 4 (1)(83) 128 170.67
12
1
2 8(1) 0.5 (8)(13) 2   2.67
12
1
3 1(3) 1.5 (1)(33) 6.75   9.00
12

Thus,

Ix = Σ(Ix)i = 182.33 in4 = 182 in4         Ans.

Ans:
 Ix = 182 in4

1039
© 2016 Pearson Education, Inc., Upper Saddle River, NJ. All rights reserved. This material is protected under all copyright laws as they currently
exist. No portion of this material may be reproduced, in any form or by any means, without permission in writing from the publisher.

10–31.

Determine the moment of inertia for the beam’s cross- y


1 in.
sectional area about the y axis.

1 in.
8 in.

3 in.
x
1 in. 10 in.

Solution
Moment of Inertia. The moments of inertia about the y axis for each segment can
be determined using the parallel axis theorem, Ix = Ix′ + Ad 2y. Referring to Fig. a,

Segment Ai(in2) (dx)i (in.) (Iy')i (in4) (Adx2)i (in4) (Iy)i (in4)
1
1 8(1) 9.5 (8)(13) 722 722.67
12
1
2 1(8) 5 (1)(83) 200 242.67
12
1
3 3(1) 0.5 (3)(13) 0.75 1.00
12

Thus,
Iy = Σ(Iy)i = 966.33 in4 = 966 in4          Ans.

Ans:
Iy = 966 in4

1040
© 2016 Pearson Education, Inc., Upper Saddle River, NJ. All rights reserved. This material is protected under all copyright laws as they currently
exist. No portion of this material may be reproduced, in any form or by any means, without permission in writing from the publisher.

*10–32.

Determine the moment of inertia Ix of the shaded area y


about the x axis.
100 mm 100 mm 150 mm

150 mm

150 mm 75 mm

x
Solution O

Moment of Inertia. The moment of inertia about the x axis for each segment can
be determined using the parallel axis theorem, Ix = Ix′ + Ad 2y. Referring to Fig. a

Segment Ai(mm2) (dy)i (mm) (Ix′)i (mm4) (Ady)2i (mm4) (Ix)i (mm4)
1
1 200(300) 150 (200)(3003) 1.35(109) 1.80(109)
12
1 1
2 (150)(300) 100 (150)(3003) 0.225(109) 0.3375(109)
2 36
3 - p(752) 150 p(754) - 0.3976(109) - 0.4225(109)
-
4

Thus,
Ix = Σ(Ix)i = 1.715(109) mm4 = 1.72(109) mm4      Ans.

Ans:
Ix = 1.72(109) mm4

1041
© 2016 Pearson Education, Inc., Upper Saddle River, NJ. All rights reserved. This material is protected under all copyright laws as they currently
exist. No portion of this material may be reproduced, in any form or by any means, without permission in writing from the publisher.

10–33.

Determine the moment of inertia Ix of the shaded area y


about the x axis.
100 mm 100 mm 150 mm

150 mm

150 mm 75 mm

x
Solution O

Moment of Inertia. The moment of inertia about the y axis for each segment can be
determined using the parallel-axis theorem, Iy = Iy′ + Ad 2x. Referring to Fig. a

Segment Ai(mm2) (dx)i (mm) Iy′(mm4) (Ad 2x)i (mm4) (Iy)i (mm4)
1
1 200(300) 100 (300)(2003) 0.6(109) 0.800(109)
12
1 1
2 (150)(300) 250 (300)(1503) 1.40625(109) 1.434375(109)
2 36
3 - p(752) 100 p(754) - 0.1767(109) - 0.20157(109)
-
4

Thus,
Iy = Σ(Iy)i = 2.033(109) mm4 = 2.03(109) mm4      Ans.

Ans:
Iy = 2.03 ( 109 ) mm4

1042
© 2016 Pearson Education, Inc., Upper Saddle River, NJ. All rights reserved. This material is protected under all copyright laws as they currently
exist. No portion of this material may be reproduced, in any form or by any means, without permission in writing from the publisher.

10–34.

Determine the moment of inertia of the beam’s cross- y


sectional area about the y axis.

150 mm
50 mm 150 mm
SOLUTION
Moment of Inertia: The dimensions and location of centroid of each segment are
shown in Fig. a. Since the y axis passes through the centroid of both segments, the x¿
moment of inertia about y axis for each segment is simply (Iy)i = (Iy¿)i. 250 mm
C
x¿
1 1
Iy = g (Iy)i = (50)(3003) + (250)(503) _
12 12 y
6 4 6 4
= 115.10(10 ) mm = 115(10 ) mm Ans.
25 mm x
25 mm

Ans:
 Iy = 115 ( 106 ) mm4

1043
© 2016 Pearson Education, Inc., Upper Saddle River, NJ. All rights reserved. This material is protected under all copyright laws as they currently
exist. No portion of this material may be reproduced, in any form or by any means, without permission in writing from the publisher.

10–35.

Determine y, which locates the centroidal axis x¿ for the y


cross-sectional area of the T-beam, and then find the
moment of inertia about the x¿ axis.

150 mm
50 mm 150 mm

SOLUTION x¿
250 mm
©yA 125(250)(50) + (275)(50)(300) C
y = =
©A 250(50) + 50(300) x¿

= 206.818 mm y

y = 207 mm Ans.
25 mm x
25 mm
1
Ix¿ = c (50)(250)3 + 50(250)(206.818 - 125)2 d
12

1
+c (300)(50)3 + 50(300)(275 - 206.818)2 d
12

I x¿ = 222(106) mm4 Ans.

Ans:
y = 207 mm
Ix′ = 222 ( 106 ) mm4

1044
© 2016 Pearson Education, Inc., Upper Saddle River, NJ. All rights reserved. This material is protected under all copyright laws as they currently
exist. No portion of this material may be reproduced, in any form or by any means, without permission in writing from the publisher.

*10–36.

Determine the moment of inertia about the x axis. y

150 mm 150 mm

20 mm

200 mm

x
C
20 mm
200 mm
Solution
20 mm
Moment of Inertia. Since the x axis passes through the centroids of the two segments,
Fig. a,

1 1
Ix = (300)(4003) - (280)(3603)
12 12
= 511.36(106) mm4

= 511(106) mm4  Ans.

Ans:
Ix = 511(106) mm4

1045
© 2016 Pearson Education, Inc., Upper Saddle River, NJ. All rights reserved. This material is protected under all copyright laws as they currently
exist. No portion of this material may be reproduced, in any form or by any means, without permission in writing from the publisher.

10–37.

Determine the moment of inertia about the y axis. y

150 mm 150 mm

20 mm

200 mm

x
C
20 mm
200 mm
Solution
20 mm
Moment of Inertia. Since the y axis passes through the centroid of the two segments,
Fig. a,

1 1
Iy = (360)(203) + (40)(3003)
12 12

= 90.24(106) mm4
= 90.2(106) mm4 Ans.

Ans:
Iy = 90.2 ( 106 ) mm4

1046
© 2016 Pearson Education, Inc., Upper Saddle River, NJ. All rights reserved. This material is protected under all copyright laws as they currently
exist. No portion of this material may be reproduced, in any form or by any means, without permission in writing from the publisher.

10–38.

Determine the moment of inertia of the shaded area about y


the x axis.
3 in.

6 in.

x
6 in. 6 in.

Solution
Moment of Inertia. The moment of inertia about the x axis for each segment can
be determined using the parallel-axis theorem, Ix = Ix′ + Ad 2y. Referring to Fig. a,

Segment Ai(in2) (dy)i (in.) (Ix′)i (in4) (Ad 2y)i (in4) (Ix)i (in4)
1
1 6(6) 3 (6)(63) 324 432.0
12
1 1
2 (6)(3) 7 (6)(33) 441 445.5
2 36
1 1
3 (6)(9) 6 (6)(93) 972 1093.5
2 36

Thus,
Ix = Σ(Ix)i = 1971 in4            Ans.

Ans:
Ix = 1971 in4

1047
© 2016 Pearson Education, Inc., Upper Saddle River, NJ. All rights reserved. This material is protected under all copyright laws as they currently
exist. No portion of this material may be reproduced, in any form or by any means, without permission in writing from the publisher.

10–39.

Determine the moment of inertia of the shaded area about y


the y axis.
3 in.

6 in.

x
6 in. 6 in.

Solution
Moment of Inertia. The moment of inertia about the y axis for each segment can
be determined using the parallel-axis theorem, Iy = Iy′ + Ad 2x. Referring to Fig. a,

Segment Ai(in2) (dx)i (in.) (Iy′)i (in4) (Ad 2x)i (in4) (Iy)i (in4)
1
1 6(6) 3 (6)(63) 324 432.0
12
1 1
2 (6)(3) 4 (3)(63) 144 162.0
2 36
1 1
3 (9)(6) 8 (9)(63) 1728 1782.0
2 36

Thus,

Iy = Σ(Iy)i = 2376 in4            Ans.

Ans:
Iy = 2376 in4

1048
© 2016 Pearson Education, Inc., Upper Saddle River, NJ. All rights reserved. This material is protected under all copyright laws as they currently
exist. No portion of this material may be reproduced, in any form or by any means, without permission in writing from the publisher.

*10–40.

Determine the distance y to the centroid of the beam’s y y¿


cross-sectional area; then find the moment of inertia about
the centroidal x′ axis.
1 in. 1 in.
3 in. 3 in.

4 in.
C
x¿
y
x
Solution 1 in.
Centroid. Referring to Fig. a, the areas of the segments and their respective centroids
are tabulated below.

∼ ∼A(in3)
Segment A(in2) y (in.) y


1 6(1) 0.5 3.00


2 2(4) 2 16.00
Σ 14.0 19.00

Σ∼
yA 19.00 in3
Thus y = = = 1.357 in. = 1.36 in. Ans.
ΣA 14.0 in2
Moment of Inertia. The moment of inertia about the x′ axis for each segment can
be determined using the parallel axis theorem, Ix′ = Ix′ + Ad 2y. Referring to Fig. b,

Segment Ai(in2) (dy)i (in.) (Ix′)i (in4) (Ad 2y)i (in4) (Ix′)i (in4)
1
1 6(1) 0.8571 (6)(13) 4.4082 4.9082
12
1
2 2(4) 0.6429 (2)(43) 3.3061 13.9728
12

Thus,
Ix′ = Σ(Ix′)i = 18.88 in4 = 18.9 in4         Ans.

Ans:
y = 1.36 in.
Ix′ = 18.9 in4

1049
© 2016 Pearson Education, Inc., Upper Saddle River, NJ. All rights reserved. This material is protected under all copyright laws as they currently
exist. No portion of this material may be reproduced, in any form or by any means, without permission in writing from the publisher.

10–41.

Determine the moment of inertia for the beam’s cross- y y¿


sectional area about the y axis.
1 in. 1 in.
3 in. 3 in.

4 in.
C
x¿
y
x
Solution 1 in.
Moment of Inertia. The moment of inertia about the y axis for each segment can
be determined using the parallel-axis theorem, Iy = Iy′ + Ad 2x. Referring to Fig. a,

Segment Ai(in2) (dx)i (in.) (Iy′)i (in4) (Ad 2x)i (in4) (Iy)i (in4)
1
1 4(1) 0.5 (4)(13) 1.00 1.3333
12
1
2 1(6) 4 (1)(63) 96.0 114.00
12
1
3 4(1) 7.5 (4)(13) 225.0 225.33
12

Thus,
    Iy = Σ(Iy)i = 340.67 in4 = 341 in4           Ans.

Ans:
Iy = 341 in4

1050
© 2016 Pearson Education, Inc., Upper Saddle River, NJ. All rights reserved. This material is protected under all copyright laws as they currently
exist. No portion of this material may be reproduced, in any form or by any means, without permission in writing from the publisher.

10–42.

Determine the moment of inertia of the beam’s cross- y y¿


sectional area about the x axis.
30 mm

30 mm
70 mm

140 mm x
C
SOLUTION
30 mm
y
1 x¿
Ix = (170)(30)3 + 170(30)(15)2
12
30 mm 170 mm
1 x
+ (30)(170)3 + 30(170)(85)2
12

1
+ (100)(30)3 + 100(30)(185)2
12

Ix = 154(106) mm4 Ans.

Ans:
Ix = 154 ( 106 ) mm4

1051
© 2016 Pearson Education, Inc., Upper Saddle River, NJ. All rights reserved. This material is protected under all copyright laws as they currently
exist. No portion of this material may be reproduced, in any form or by any means, without permission in writing from the publisher.

10–43.

Determine the moment of inertia of the beam’s cross- y y¿


sectional area about the y axis.
30 mm

30 mm
SOLUTION 70 mm
_
1 140 mm x
Iy = (30)(170)3 + 30(170)(115)2 C
12
1 30 mm _
+ (170)(30)3 + 30(170)(15)2 y
12 x¿
1
+ (30)(100)3 + 30(100)(50)2 30 mm 170 mm
12
x
Iy = 91.3(106) mm4 Ans.

Ans:
Iy = 91.3 ( 106 ) mm4

1052
© 2016 Pearson Education, Inc., Upper Saddle River, NJ. All rights reserved. This material is protected under all copyright laws as they currently
exist. No portion of this material may be reproduced, in any form or by any means, without permission in writing from the publisher.

*10–44.

Determine the distance y to the centroid C of the beam’s y y¿


cross-sectional area and then compute the moment of
inertia Ix¿ about the x¿ axis. 30 mm

30 mm
70 mm

140 mm x
C
SOLUTION
30 mm
170(30)(15) + 170(30)(85) + 100(30)(185) y
y = x¿
170(30) + 170(30) + 100(30)
30 mm 170 mm
= 80.68 = 80.7 mm Ans. x

1
Ix¿ = c (170)(30)3 + 170(30)(80.68 - 15)2 d
12

1
+c (30)(170)3 + 30(170)(85 - 80.68)2 d
12

1
+ (100)(30)3 + 100(30)(185 - 80.68)2
12

Ix¿ = 67.6(106) mm4 Ans.

Ans:
-
y = 80.7 mm
-
Ix′ = 67.6(106) mm4

1053
© 2016 Pearson Education, Inc., Upper Saddle River, NJ. All rights reserved. This material is protected under all copyright laws as they currently
exist. No portion of this material may be reproduced, in any form or by any means, without permission in writing from the publisher.

10–45.

Determine the distance x to the centroid C of the beam’s y y¿


cross-sectional area and then compute the moment of
inertia Iy¿ about the y¿ axis. 30 mm

30 mm
70 mm

140 mm x
C
SOLUTION
30 mm
170(30)(115) + 170(30)(15) + 100(30)(50) y
x = x¿
170(30) + 170(30) + 100(30)
30 mm 170 mm
= 61.59 = 61.6 mm Ans. x

1
Iy¿ = c (30)(170)3 + 170(30)(115 - 61.59)2 d
12

1
+c (170)(30)3 + 30(170)(15 - 61.59)2 d
12

1
+ (30)(100)3 + 100(30)(50 - 61.59)2
12

Iy¿ = 41.2(106) mm4 Ans.

Ans:
x = 61.6 mm
Iy= = 41.2 ( 106 ) mm4

1054
© 2016 Pearson Education, Inc., Upper Saddle River, NJ. All rights reserved. This material is protected under all copyright laws as they currently
exist. No portion of this material may be reproduced, in any form or by any means, without permission in writing from the publisher.

10–46.

Determine the moment of inertia for the shaded area about y


the x axis.
6 in.

3 in.

3 in.

2 in.
3 in.
x
3 in. 3 in.

Solution
Moment of Inertia. The moment of inertia about the x axis for each segment can
be determined using the parallel-axis theorem, Ix = Ix′ + Ad 2y. Referring to Fig. a,

Segment Ai(in2) (dy)i (in.) (Ix′)i (in4) (Ad 2y)i (in4) (Ix)i (in4)
1
1 6(6) 3 (6)(63) 324 432.00
12
1 1
2 (6)(3) 7 (6)(33) 441 445.50
2 36
1 1
3 (6)(9) 6 (6)(93) 972 1093.50
2 36
p(24)
4 - p(22) 3 - -36p -40p
4

Thus,
Ix = Σ(Ix)i = 1845.34 in4 = 1845 in4 Ans.

Ans:
Ix = 1845 in4

1055
© 2016 Pearson Education, Inc., Upper Saddle River, NJ. All rights reserved. This material is protected under all copyright laws as they currently
exist. No portion of this material may be reproduced, in any form or by any means, without permission in writing from the publisher.

10–47.

Determine the moment of inertia for the shaded area about y


the y axis.
6 in.

3 in.

3 in.

2 in.
3 in.
x
3 in. 3 in.

Solution
Moment of Inertia. The moment of inertia about the x axis for each segment can
be determined using the parallel-axis theorem, Iy = Iy′ + Ad 2x. Referring to Fig. a,

Segment Ai(in2) (dx)i (in.) (Ix′)i (in4) (Ad 2x)i (in4) (Iy)i (in4)
1
1 6(6) 3 (6)(63) 324 432.0
12
1 1
2 (6)(3) 2 (3)(63) 36 54.0
2 36
1 1
3 (9)(6) 2 (9)(63) 108 162.0
2 36
p(24)
4 - p(22) 3 - -36p -40p
4

Thus,
   Iy = Σ(Iy)i = 522.34 in4 = 522 in4         Ans.

Ans:
Iy = 522 in4

1056
© 2016 Pearson Education, Inc., Upper Saddle River, NJ. All rights reserved. This material is protected under all copyright laws as they currently
exist. No portion of this material may be reproduced, in any form or by any means, without permission in writing from the publisher.

*10–48.

Determine the moment of inertia of the parallelogram y y'


about the x′ axis, which passes through the centroid C of
the area.

C
a x'
θ
x
SOLUTION b

h = a sin u

1 1 1 3
Ix′ = bh3 = (b)(a sin u)3 = a b sin3 u Ans.
12 12 12

Ans:
- 1 3
Ix′ = a b sin3 u
12

1057
© 2016 Pearson Education, Inc., Upper Saddle River, NJ. All rights reserved. This material is protected under all copyright laws as they currently
exist. No portion of this material may be reproduced, in any form or by any means, without permission in writing from the publisher.

10–49.

Determine the moment of inertia of the parallelogram y y'


about the y′ axis, which passes through the centroid C of
the area.

C
a x'
θ
x
SOLUTION b

b - a cos u 1
x = a cos u + = (a cos u + b)
2 2

2
Iy′ = 2 J (a sin u)(a cos u)3 + (a sin u)(a cos u) a + cos u - a cos u b R
1 1 b a 2
36 2 2 2 3

1
+ (a sin u)(b - a cos u)3
12

ab sin u 2
= ( b + a2 cos2 u ) Ans.
12

Ans:
ab sin u 2
Iy′ = ( b + a2 cos2 u )
12

1058
© 2016 Pearson Education, Inc., Upper Saddle River, NJ. All rights reserved. This material is protected under all copyright laws as they currently
exist. No portion of this material may be reproduced, in any form or by any means, without permission in writing from the publisher.

10–50.

Locate the centroid y of the cross section and determine the


moment of inertia of the section about the x¿ axis.

0.4 m x'
–y
0.05 m

0.3 m
0.2 m 0.2 m 0.2 m 0.2 m
SOLUTION
Centroid: The area of each segment and its respective centroid are tabulated below.

Segment A (m2) y (m) yA (m3)


1 0.3(0.4) 0.25 0.03
1
2 2 10.4210.42 0.1833 0.014667

3 1.1(0.05) 0.025 0.001375

© 0.255 0.046042

Thus,

©yA 0.046042
y = = = 0.1806 m = 0.181 m Ans.
©A 0.255

Moment of Inertia: The moment of inertia about the x¿ axis for each segment can be
determined using the parallel-axis theorem Ix¿ = Ix¿ + Ad2y.

Segment Ai (m2) (dy)i (m) (Ix¿)i (m4) (Ad 2y)i (m4) (Ix¿)i (m4)
1
1 0.3(0.4) 0.06944 3
12 10.3210.4 2 0.5787110-32 2.1787110-32
1 1
2 2 10.4210.42 0.002778 3
36 10.4210.4 2 0.6173110-62 0.7117110-32
1
3 1.1(0.05) 0.1556 3
12 11.1210.05 2 1.3309110-32 1.3423110-32

Thus,

Ix¿ = © Ix¿ i = 4.233 10 -3 m4 = 4.23 10-3 m4 Ans.

Ans:
y = 0.181 m
Ix′ = 4.23(10-3) m4

1059
© 2016 Pearson Education, Inc., Upper Saddle River, NJ. All rights reserved. This material is protected under all copyright laws as they currently
exist. No portion of this material may be reproduced, in any form or by any means, without permission in writing from the publisher.

10–51.

Determine the moment of inertia for the beam’s cross-


sectional area about the x¿ axis passing through the centroid 100 mm
100 mm
C of the cross section.
25 mm

200 mm
200 mm
45

SOLUTION 45
C 45

1 1 1 141.4 2 x¿
(200)(332.8)3 + 4 c (141.4)(141.4)3 + a (141.4)(141.4) b a b d
45
Ix¿ =
12 36 2 3
25 mm
1 1
- 2 c (200)4 a - sin90° b d
p
4 4 2

= 520(106) mm4 Ans.

Ans:
Ix= = 520 ( 106 ) mm4

1060
© 2016 Pearson Education, Inc., Upper Saddle River, NJ. All rights reserved. This material is protected under all copyright laws as they currently
exist. No portion of this material may be reproduced, in any form or by any means, without permission in writing from the publisher.

*10–52.

Determine the moment of inertia of the area about y


the x axis.
3 in. 3 in.

6 in.
SOLUTION

Ix = B (6)(10)3 + 6(10)(5)2 R - B (3)(6)3 + a b(3)(6)(8)2 R


1 1 1
12 36 2 2 in.
4 in.
- B p (2)4 + p(2)2(4)2 R = 1.19(103) in4
1
Ans.
4 x

Ans:
Ix = 1.19(103) in4

1061
© 2016 Pearson Education, Inc., Upper Saddle River, NJ. All rights reserved. This material is protected under all copyright laws as they currently
exist. No portion of this material may be reproduced, in any form or by any means, without permission in writing from the publisher.

10–53.

Determine the moment of inertia of the area about the y


y axis.
3 in. 3 in.

SOLUTION
1 1 1
Iy = c (10)(6)3 + 6(10)(3)2 d - c (6)(3)3 + a b 6(3)(5)2 d 6 in.
12 36 2
1
- c p(2)4 + p(2)2(3)2 d = 365 in4 Ans.
4
2 in.
4 in.

Ans:
Iy = 365 in4

1062
© 2016 Pearson Education, Inc., Upper Saddle River, NJ. All rights reserved. This material is protected under all copyright laws as they currently
exist. No portion of this material may be reproduced, in any form or by any means, without permission in writing from the publisher.

10–54.
y

Determine the product of inertia of the thin strip of area t


with respect to the x and y axes. The strip is oriented at an
angle u from the x axis. Assume that t V l.

SOLUTION
l 1 u
2 x
lxy = xydA = (s cos u)(s sin u)tds = sin u cos ut s ds
LA L0 L0

1 3
= l t sin 2u Ans.
6

Ans:
1 3
Ixy = tl sin 2u
3

1063
© 2016 Pearson Education, Inc., Upper Saddle River, NJ. All rights reserved. This material is protected under all copyright laws as they currently
exist. No portion of this material may be reproduced, in any form or by any means, without permission in writing from the publisher.

10–55.

Determine the product of inertia of the shaded area with y


respect to the x and y axes.

y  1 x3
SOLUTION 3 in. 9

Differential Element: The area of the differential element parallel to the y axis
1
shown shaded in Fig. a is dA = y dx = x3 dx. The coordinates of the centroid of
9 x
3 in.
y 1 3
this element are xc = x and yc = = x . Thus, the product of inertia of this
2 18
element with respect to the x and y axes is

''
dIxy = dIx¿y¿ + dAx y
1 1
= 0 + a x3dx b(x) a x3 b
9 18
1 7
= x dx
162
Product of Inertia: Performing the integration, we have

3 in 3 in.
1 7 1
Ixy = dIxy = x dx = (x8) 2 = 5.06 in4 Ans.
L L0 162 1296 0

Ans:
Ixy = 5.06 in4

1064
© 2016 Pearson Education, Inc., Upper Saddle River, NJ. All rights reserved. This material is protected under all copyright laws as they currently
exist. No portion of this material may be reproduced, in any form or by any means, without permission in writing from the publisher.

*10–56.

Determine the product of inertia for the shaded portion of y


the parabola with respect to the x and y axes.
100 mm

200 mm
1 2
y x
50
SOLUTION
1
Differential Element: Here, x = 250y2 . The 1
area of the differential element x
parallel to the x axis is dA = 2xdy = 2 250y2 dy. The coordinates of the centroid
for this element are x = 0, y = y. Then the product of inertia for this element is

dIxy = dIx¿y¿ + dAx y

= 0 + ¢ 2 250y2 dy ≤ (0)(y)
1

= 0

Product of Inertia: Performing the integration, we have

Ixy = dIxy = 0 Ans.


L

Note: By inspection, Ixy = 0 since the shaded area is symmetrical about the y axis.

Ans:

L
Ixy = dIxy = 0

1065
© 2016 Pearson Education, Inc., Upper Saddle River, NJ. All rights reserved. This material is protected under all copyright laws as they currently
exist. No portion of this material may be reproduced, in any form or by any means, without permission in writing from the publisher.

10–57.

Determine the product of inertia of the shaded area with y y¿


respect to the x and y axes, and then use the parallel-axis
theorem to find the product of inertia of the area with
respect to the centroidal x¿ and y¿ axes.
y2 x
2m
x¿
C
SOLUTION x

Differential Element: The area of the differential element parallel to the y axis 4m
shown shaded in Fig. a is dA = y dx = x 1>2 dx. The coordinates of the centroid of
' ' y 1 1>2
this element are x = x and y = = x Thus, the product of inertia of this
2 2
element with respect to the x and y axes is

~~
dIxy = dIx¿y¿ + dAx y

= 0 + A x1>2 dx B (x) a x 1>2 b


1
2
1 2
=x dx
2
Product of Inertia: Performing the integration, we have

4m 4m
1 2 1
Ixy = dIxy = x dx = a x3 b 2 = 10.67 m4 = 10.7 m4 Ans.
L L0 2 6 0

Using the information provided on the inside back cover of this book, the location of
2 3
the centroid of the parabolic area is at x = 4 - (4) = 2.4 m and y = (2) = 0.75 m
5 8
2
and its area is given by A = (4)(2) = 5.333 m2. Thus,
3

Ixy = Ix¿y¿ + Adxdy


10.67 = Ix¿y¿ + 5.333(2.4)(0.75)
Ix¿y¿ = 1.07 m4 Ans.

Ans:
Ixy = 10.7 m4
Ix′y′ = 1.07 m4

1066
© 2016 Pearson Education, Inc., Upper Saddle River, NJ. All rights reserved. This material is protected under all copyright laws as they currently
exist. No portion of this material may be reproduced, in any form or by any means, without permission in writing from the publisher.

10–58.

Determine the product of inertia for the parabolic area with y


b 1/2
respect to the x and y axes. y x
a1/2

x
SOLUTION a
'
x = x

' y
y =
2

dA = y dx

xy2
dIxy = dx
2

Ixy = d Ixy
L
a
1 b2 2
= a b x dx
L0 2 a

B a b x3 R
1 b2 a
=
6 a 0

1 2 2
= a b Ans.
6

Ans:
1 2 2
Ixy = ab
6

1067
© 2016 Pearson Education, Inc., Upper Saddle River, NJ. All rights reserved. This material is protected under all copyright laws as they currently
exist. No portion of this material may be reproduced, in any form or by any means, without permission in writing from the publisher.

10–59.

Determine the product of inertia of the shaded area with y


respect to the x and y axes.

1 1
y = (a2 – x 2 )2
a

SOLUTION
Differential Element: The area of the differential element parallel to the y axis is x
dA = ydx = A a2 - x2 B dx. The coordinates of the centroid for this element are
1 1 2 O
a

= A a2 - x2 B 2. Then the product of inertia for this element is


' ' y 1 1 1
x = x, y =
2 2
''
dIxy = dIx¿y¿ + dAx y

= 0 + c A a2 - x2 B 2 dx d1x2 c a a2 - x2 b d
2
1 1 1 1 1

A x + a2x + 6ax2 - 4a2x2 - 4a2x2 B dx


1 3 3 3 1 5
=
2

Product of Inertia: Performing the integration, we have


a
1
¢ x3 + a2x + 6ax2 - 4a2x2 - 4a2x2 bdx
3 3 1 5
Ixy = dIxy =
L 2 L0

x + 2ax3 - a2x2 - a2x2 `


1 x4 a2 2 8 3 5 8 1 7 a
= +
2 4 2 5 7 0

a4
= Ans.
280

Ans:
a4
Ixy =
280

1068
© 2016 Pearson Education, Inc., Upper Saddle River, NJ. All rights reserved. This material is protected under all copyright laws as they currently
exist. No portion of this material may be reproduced, in any form or by any means, without permission in writing from the publisher.

*10–60.

Determine the product of inertia of the shaded area with y


respect to the x and y axes.

x2 + y2 = 4

2 in.

SOLUTION
x
Differential Element: Here, y = 24 - x2 . The area of the differential element 2 in.

parallel to the y axis is dA = ydx = 24 - x2dx. The coordinates of the centroid


' ' y 1
for this element are x = x, y = = 24 - x2 . Then the product of inertia for this
2 2
element is

''
dIxy = dIx¿y¿ + dAx y

A 24 - x2dx B 1x2a 24 - x2 b
1
= 0 +
2

1
= 14x - x32 dx
2

Product of Inertia: Performing the integration, we have


2in.
1
Ixy = dIxy = 14x - x32 dx
L 2 L0

1 x4 2in.
= 2x2 - = 2.00 in4 Ans.
2 4 0

Ans:
Ixy = 2.00 in4

1069
© 2016 Pearson Education, Inc., Upper Saddle River, NJ. All rights reserved. This material is protected under all copyright laws as they currently
exist. No portion of this material may be reproduced, in any form or by any means, without permission in writing from the publisher.

10–61.

Determine the product of inertia of the shaded area with y


respect to the x and y axes.

1 in.
y = 0.25x 2
x
2 in.
SOLUTION
1
Differential Element: Here, x = 2y 2. The area of the differential element parallel
1
to the x axis is dA = xdy = 2y 2 dy. The coordinates of the centroid for this element
x 1
are x = = y 2, y = y. Then the product of inertia for this element is
2

dIxy = dIx′y′ + dAx y

= 0 + a2y 2 dyb ( y2 ) (y)


1 1

= 2y2 dy

Product of Inertia: Performing the integration, we have

l in.
2 3 1 in.
Ixy = dIxy = 2y2 dy = y ` = 0.667 in4 Ans.
L L0 3 0

Ans:
Ixy = 0.667 in4

1070
© 2016 Pearson Education, Inc., Upper Saddle River, NJ. All rights reserved. This material is protected under all copyright laws as they currently
exist. No portion of this material may be reproduced, in any form or by any means, without permission in writing from the publisher.

10–62.

Determine the product of inertia for the beam’s cross- y


sectional area with respect to the x and y axes. 1 in.

1 in.
8 in.

SOLUTION 3 in.
1 in. x
Ixy = 0.5(4)(8)(1) + 6(0.5)(10)(1) + 11.5(1.5)(3)(1) 12 in.
4
= 97.8 in Ans.

Ans:
Ixy = 97.8 in4

1071
© 2016 Pearson Education, Inc., Upper Saddle River, NJ. All rights reserved. This material is protected under all copyright laws as they currently
exist. No portion of this material may be reproduced, in any form or by any means, without permission in writing from the publisher.

10–63.

Determine the moments of inertia for the shaded area with y


respect to the u and v axes.
0.5 in.
v
u

SOLUTION
30 0.5 in.
5 in. x
Moment and Product of Inertia about x and y Axes: Since the shaded area is 0.5 in.
symmetrical about the x axis, Ixy = 0.

1 1
Ix = (1)(53) + (4)(13) = 10.75 in4
12 12 4 in.
1 in.
1 1
Iy = (1)(43) + 1(4)(2.5)2 + (5)(13) = 30.75 in4
12 12

Moment of Inertia about the Inclined u and v Axes: Applying Eq. 10-9 with
u = 30°, we have
I x + Iy Ix - I y
Iu = + cos 2u - Ixy sin 2u
2 2

10.75 + 30.75 10.75 - 30.75


= + cos 60° - 0(sin 60°)
2 2

= 15.75 in4 Ans.


Ix + I y I x - Iy
Iv = - cos 2u + Ixy sin 2u
2 2

10.75 + 30.75 10.75 - 30.75


= - cos 60° + 0(sin 60°)
2 2

= 25.75 in4 Ans.

Ans:
Iu = 15.75 in4
Iu = 25.75 in4

1072
© 2016 Pearson Education, Inc., Upper Saddle River, NJ. All rights reserved. This material is protected under all copyright laws as they currently
exist. No portion of this material may be reproduced, in any form or by any means, without permission in writing from the publisher.

*10–64.

Determine the product of inertia for the beam’s cross- y


v
sectional area with respect to the u and v axes.
150 mm
150 mm

20
SOLUTION x
C
Moments of inertia Ix and Iy 20 mm
200 mm
1 1
Ix = (300)(400)3 - (280)(360)3 = 511.36(10)6 mm4
12 12 20 mm

1 1
Iy = 2 c (20)(300)3 d + (360)(20)3 = 90.24(10)6 mm4
12 12

The section is symmetric about both x and y axes; therefore Ixy = 0.


I x - Iy
Iuv = sin 2u + Ixy cos 2u
2

511.36 - 90.24
= a sin 40° + 0 cos 40°b 106
2

= 135(10)6 mm4 Ans.

Ans:
Iuv = 135(10)6 mm4

1073
© 2016 Pearson Education, Inc., Upper Saddle River, NJ. All rights reserved. This material is protected under all copyright laws as they currently
exist. No portion of this material may be reproduced, in any form or by any means, without permission in writing from the publisher.

10–65.

Determine the product of inertia for the shaded area with y


respect to the x and y axes.

2 in. 2 in.

2 in.

SOLUTION 1 in.
''
Ixy = ©(Ix¿y¿ + x y A) = [0 + 2(3)(4)(6)] - C 0 + 2(4)(p)(1)2 D 4 in.

= 119 in4 Ans.


x

Ans:
Ixy = 119 in4

1074
© 2016 Pearson Education, Inc., Upper Saddle River, NJ. All rights reserved. This material is protected under all copyright laws as they currently
exist. No portion of this material may be reproduced, in any form or by any means, without permission in writing from the publisher.

10–66.

Determine the product of inertia of the cross-sectional area y


with respect to the x and y axes.

100 mm

20 mm

SOLUTION 400 mm

Product of Inertia: The area for each segment, its centroid and product of inertia
with respect to x and y axes are tabulated below. C
x

Segment Ai (mm2) (dx)i (mm) (dy)i (mm) (Ixy)i (mm4)


400 mm
1 100(20) 60 410 49.211062
2 840(20) 0 0 0
20 mm
3 100(20) - 60 -410 49.211062 100 mm 20 mm

Thus,

Ixy = ©1Ixy2i = 98.411062mm4 Ans.

Ans:
Ixy = 98.4 ( 106 ) mm4

1075
© 2016 Pearson Education, Inc., Upper Saddle River, NJ. All rights reserved. This material is protected under all copyright laws as they currently
exist. No portion of this material may be reproduced, in any form or by any means, without permission in writing from the publisher.

10–67.

Determine the location (x, y) of the centroid C of the y y¿


angle’s cross-sectional area, and then compute the product
of inertia with respect to the x′ and y′ axes.
x
18 mm

150 mm

C x¿
y
x
150 mm 18 mm

SOLUTION
Centroid:
ΣxA 9(18)(150) + 84(18)(132)
x = = = 44.11 mm = 44.1 mm Ans.
ΣA 18(150) + 18(132)

ΣyA 75(18)(150) + 9(18)(132)


y = = = 44.11 mm = 44.1 mm Ans.
ΣA 18(150) + 18(132)

Product of inertia about x′ and y′ axes:

Ix′y′ = 18(150)( - 35.11)(30.89) + 18(132)(39.89)( -35.11)

= -6.26(106) mm4 Ans.

Ans:
x = y = 44.1 mm
Ix=y= = -6.26 ( 106 ) mm4

1076
© 2016 Pearson Education, Inc., Upper Saddle River, NJ. All rights reserved. This material is protected under all copyright laws as they currently
exist. No portion of this material may be reproduced, in any form or by any means, without permission in writing from the publisher.

*10–68.

Determine the distance y to the centroid of the area and y


then calculate the moments of inertia Iu and Iv for the v
channel’s cross-sectional area. The u and v axes have their
origin at the centroid C. For the calculation, assume all
corners to be square.
10 mm 10 mm
u

50 mm C 20 x
10 mm y
SOLUTION
150 mm 150 mm
300(10)(5) + 2[(50)(10)(35)]
y = = 12.5 mm Ans.
300(10) + 2(50)(10)

1
Ix = c (300)(10)3 + 300(10)(12.5 - 5)2 d
12

1
+ 2c (10)(50)3 + 10(50)(35 - 12.5)2 d
12

= 0.9083(106) mm4

1 1
Iy = (10)(300)3 + 2 c (50)(10)3 + 50(10)(150 - 5)2 d
12 12

= 43.53(106) mm4

Ixy = 0 (By symmetry)


Ix + Iy Ix - Iy
Iu = + cos 2u - Ixy sin 2u
2 2

0.9083(106) + 43.53(106) 0.9083(106) - 43.53(106)


= + cos 40° - 0
2 2

= 5.89(106) mm4 Ans.


Ix + Iy Ix - Iy
Iv = - cos 2u + Ixy sin 2u
2 2

0.9083(106) + 43.53(106) 0.9083(106) - 43.53(106)


= - cos 40°+0
2 2

= 38.5(106) mm4 Ans.

1077
© 2016 Pearson Education, Inc., Upper Saddle River, NJ. All rights reserved. This material is protected under all copyright laws as they currently
exist. No portion of this material may be reproduced, in any form or by any means, without permission in writing from the publisher.

10–69.

Determine the moments of inertia Iu, Iv and the product of y


inertia Iuv for the beam’s cross-sectional area. Take u = 45°.
v u

16 in.

u
SOLUTION x
O
1 1 8 in. 8 in. 2 in.
Ix = (20)(2)3 + 20(2)(1)2 + (4)(16)3 + 4(16)(8)2 2 in. 2 in.
12 12

= 5.515(103) in4

1 1
Iy = (2)(20)3 + (16)(4)3
12 12

= 1.419(103) in4

Ixy = 0
Ix + Iy Ix - Iy
Iu = + cos 2u - Ixy sin 2u
2 2

5.515 + 1.419 3 5.515 - 1.419 3


= (10 ) + (10 ) cos 90° - 0
2 2

= 3.47(103) in4 Ans.

Iv = 3.47(103) in4 Ans.


Ix - Iy
Iuv = sin 2u + Ixy cos 2u
2

5.515 - 1.419 3
= (10 ) sin 90° + 0
2

= 2.05(103) in4 Ans.

Ans:
Iu = 3.47 ( 103 ) in4
Iv = 3.47 ( 103 ) in4
Iuv = 2.05 ( 103 ) in4

1078
© 2016 Pearson Education, Inc., Upper Saddle River, NJ. All rights reserved. This material is protected under all copyright laws as they currently
exist. No portion of this material may be reproduced, in any form or by any means, without permission in writing from the publisher.

10–70.

Determine the moments of inertia Iu, Iv and the product of y


inertia Iuv for the rectangular area. The u and v axes pass
v
through the centroid C.

30
x
C 30 mm

Solution 120 mm

Moment And Product of Inertia About x and y Axes. Since the rectangular area is
symmetrical about the x and y axes, Ixy = 0.

1 1
Ix = (120)(303) = 0.270(106) mm4  Iy = (30)(1203) = 4.32(106) mm4
12 12

Moment And Product of Inertia About The Inclined u and v Axes. With u = 30°,
Ix + Iy Ix - Iy
Iu = + cos 2u - Ixy sin 2u
2 2

0.270 + 4.32 0.27 - 4.32


= c + cos 60° - 0 sin 60° d (106)
2 2

= 1.2825(106) mm4 = 1.28(106) mm4 Ans.

Ix + Iy Ix - Iy
Iv = - cos 2u + Ixy sin 2u
2 2

0.27 + 4.32 0.27 - 4.32


= c - cos 60° + 0 sin 60° d (106)
2 2

= 3.3075(106) mm4 = 3.31(106) mm4 Ans.

Ix - Iy
Iuv = sin 2u + Ixy cos 2u
2

0.270 - 4.32
= c sin 60° + 0 cos 60° d (106)
2

= - 1.7537(106) mm4 = - 1.75(106) mm4 Ans.

Ans:
Iu = 1.28(106) mm4
Iv = 3.31(106) mm4
Iuv = - 1.75(106) mm4

1079
© 2016 Pearson Education, Inc., Upper Saddle River, NJ. All rights reserved. This material is protected under all copyright laws as they currently
exist. No portion of this material may be reproduced, in any form or by any means, without permission in writing from the publisher.

10–71.

Solve Prob. 10–70 using Mohr’s circle. Hint: To solve, find y


the coordinates of the point P(Iu, Iuv) on the circle, measured
v
counterclockwise from the radial line OA. (See Fig. 10–19.)
The point Q(Iv, -Iuv) is on the opposite side of the circle.
u

30
x
C 30 mm

Solution 120 mm

Moment And Product of Inertia About x And y Axes. Since the rectangular Area is
symmetrical about the x and y axes, Ixy = 0.

1 1
Ix = (120)(303) = 0.270(106) mm4  Iy = (30)(1203) = 4.32(106) mm4
12 12
Construction of The Circle. The Coordinates of center O of the circle are
Ix + Iy 0.270 + 4.32
a , 0b = a , 0b(106) = (2.295, 0)(106)
2 2
And the reference point A is

(Ix, Ixy) = (0.270, 0)(106)


Thus, the radius of the circle is

R = OA = 1 2(2.295 - 0.27)2 + 02 2 (106) = 2.025(106) mm4

Using these Results, the circle shown in Fig. a can be constructed. Rotate radial line
OA counterclockwise 2u = 60° to coincide with radial line OP where coordinate of
point P is 1 Iu, Iuv 2. Then

Iu = (2.295 - 2.025 cos 60°)(106) = 1.2825(106) mm4 = 1.28(106) mm4   Ans.


Iuv = - 2.025(106) sin 60° = - 1.7537(106) mm4 = - 1.75(106) mm4   Ans.

Iv is represented by the coordinate of point Q. Thus,

Iv = (2.295 + 2.025 cos 60°)(106) = 3.3075(106) mm4 = 3.31(106) mm4 Ans.

Ans:
Iu = 1.28 ( 106 ) mm4
Iuv = - 1.75 ( 106 ) mm4
Iv = 3.31 ( 106 ) mm4

1080
© 2016 Pearson Education, Inc., Upper Saddle River, NJ. All rights reserved. This material is protected under all copyright laws as they currently
exist. No portion of this material may be reproduced, in any form or by any means, without permission in writing from the publisher.

*10–72.

Determine the directions of the principal axes having an y


origin at point O, and the principal moments of inertia for
the triangular area about the axes.

6 in.

x
Solution O
9 in.
Moment And Product of Inertia About x And y Axes. Using the parallel-axis theorem,
2
1 1 1
Ix = Ix′ + Ad 2y  Ix = (9) ( 63 ) + c (9)(6) d c (6) d = 162 in4
36 2 3
2
1 1 2
Iy = Iy′ + Ad 2x  Iy = (6) ( 93 ) + c (6)(9) d c (9) d = 1093.5 in4
36 2 3
Using the result of Example 10–6,

b2h2 ( 92 )( 62 )
Ixy =
= = 364.5 in.4
8 8
Principal Moments of Inertia.

Ix + Iy Ix - Iy 2
I max = { Aa b + I 2xy
min 2 2
162 + 1093.5 162 - 1093.5 2
= { a b + 364.5
2
2 A 2

= 627.75 { 591.42

Imax = 1219.17 in4 = 1219 in4 Ans.

Imin = 36.33 in4 = 36.3 in4 Ans.

The orientation of the principal axes can now be determined


- Ixy - 364.5
tan 2up = = = 0.7826
(Ix - Iy)>2 (162 - 1093.5)>2
2uP = 38.04°  And   -141.95°
up = 19.02°  And   -70.98°
Substitute up = - 70.98° into the equation for Iu,

Ix + Iy Ix - Iy
Iu = + cos 2u - Ixy sin 2u
2 2
162 + 1093.5 162 - 1093.5
= + cos ( - 141.95°) - 364.5 sin ( -141.95°)
2 2
= 1219.17 in4
Thus, (uP)1 = - 71.0° (up)2 = 19.0° Ans.

Ans:
Imax = 1219 in4
Imin = 36.3 in4
(uP)1 = - 71.0°
(up)2 = 19.0°

1081
© 2016 Pearson Education, Inc., Upper Saddle River, NJ. All rights reserved. This material is protected under all copyright laws as they currently
exist. No portion of this material may be reproduced, in any form or by any means, without permission in writing from the publisher.

10–73.

Solve Prob. 10–72 using Mohr’s circle.


y

6 in.

Solution x
O
Moment And Product of Inertia about x and y Axes. Using the parallel-axis theorem, 9 in.

2
1 1 1
  Ix = Ix′ + Ad 2y;  Ix = (9)(63) + c (9)(6) d c (6) d = 162 in4
36 2 3
2
1 1 2
  Iy = Iy′ + Ad 2x;  Iy = (6)(93) + c (6)(9) d c (9) d = 1093.5 in4
36 2 3
Using these results of Example 10–6

b2h2 (92)(62)
  Ixy = = = 364.5 in4
8 8
Construction of The Circle. The coordinates of center O of the circle are
Ix + Iy 162 + 1093.5
  a , 0b = a , 0b = (627.75, 0)
2 2
And the reference point A is
  (Ix, Ixy) = (162, 364.5)

Thus, the radius of the circle is

  R = OA = 2(627.75 - 162)2 + (0 - 364.5)2 = 591.42

Using these results the circle shown in Fig. a can be constructed. Here, the coordinates
of points B and C represent Imin and Imax respectively. Thus

  Imax = 627.75 + 591.42 = 1219.17 in4 = 1219 in4 Ans.


4 4
  Imin = 627.75 - 591.42 = 36.33 in = 36.3 in  Ans.

1082
© 2016 Pearson Education, Inc., Upper Saddle River, NJ. All rights reserved. This material is protected under all copyright laws as they currently
exist. No portion of this material may be reproduced, in any form or by any means, without permission in writing from the publisher.

10–73. Continued

The orientation of principal axes can be determined from the geometry of the
shaded triangle on the circle.

364.5
  tan 2(up)2 = = 0.7826
627.75 - 162
2(up)2 = 38.04°
(up)2 = 19.02° = 19.0° d Ans.

And

2(up)1 = 180° - 2(up)2 = 180° - 38.04° = 141.95°


(up)1 = 70.98° = 71.0° b Ans.

Ans:
Imax = 1219 in4
Imin = 36.3 in4
(up)2 = 19.0° d
(up)1 = 71.0° b

1083
© 2016 Pearson Education, Inc., Upper Saddle River, NJ. All rights reserved. This material is protected under all copyright laws as they currently
exist. No portion of this material may be reproduced, in any form or by any means, without permission in writing from the publisher.

10–74.

Determine the orientation of the principal axes having an y


origin at point C, and the principal moments of inertia of
the cross section about these axes.

100 mm
80 mm
C
x
80 mm
10 mm 100 mm

Solution
Moment and Product of Inertia About x and y Axes. Using the parallel-axis
theorem by referring to Fig. a 10 mm

1 1
Ix = Σ ( Ix′ + Ad 2y ) ;  Ix = (140)(103) + 2 c (10)(1003) + 10(100)(452) d
12 12
= 5.7283 (106) mm4

1 1
Iy = Σ ( Iy′ + Ad 2x ) ;  Iy = (10)(1403) + 2 c (100)(103) + 100(10)(752) d
12 12
= 13.5533 (106) mm4

Ixy = Σ(Ix′y′ + Ad x d y);  Ixy = 0 + 30 + 10(100)( - 75)(45) 4

+ [0 + 10(100)(75)( - 45)] = - 6.75(106) mm4

Principal moments of Inertia.

Ix + Iy Ix + Iy 2
I max = { a b + I 2xy
min 2 C 2

5.7283 + 13.5533 5.7283 - 13.5533 2


= £ { a b + ( -6.75)2 § (106)
2 C 2

= (9.6408 { 7.8019)(106)

Imax = 17.44(106) mm4 = 17.4(106) mm4 Ans.

Imin = 1.8389(106) mm4 = 1.84(106) mm4 Ans.

1084
© 2016 Pearson Education, Inc., Upper Saddle River, NJ. All rights reserved. This material is protected under all copyright laws as they currently
exist. No portion of this material may be reproduced, in any form or by any means, without permission in writing from the publisher.

10–74. Continued

The orientation of the principal axes can now be determined


- Ixy - ( - 6.75)
tan 2up = = = - 1.7252
(Ix - Iy)>2 (5.7283 - 13.5533)>2
2up = - 59.90°  and  120.10°
up = - 29.95°  and   60.04°

Substitute up = 60.05° into the equation for Iu,


Ix + Iy Ix - Iy
Iu = + cos 2u - Ixy sin 2u
2 2
5.7283 + 13.5533 5.7283 - 13.5533
= c + cos 120.10° - ( -6.75) sin 120.10° d (106)
2 2
= 17.44(106) mm4

Thus,

(up)1 = 60.0°  (up)2 = - 30.0° Ans.

Ans:
Imax = 17.4 ( 106 ) mm4
Imin = 1.84 ( 106 ) mm4
(up)1 = 60.0°
(up)2 = - 30.0°

1085
© 2016 Pearson Education, Inc., Upper Saddle River, NJ. All rights reserved. This material is protected under all copyright laws as they currently
exist. No portion of this material may be reproduced, in any form or by any means, without permission in writing from the publisher.

10–75.

Solve Prob. 10–74 using Mohr’s circle. y

100 mm
80 mm
C
x
80 mm
10 mm 100 mm

Solution
Moment and Product of Inertia About x and y Axes. Using the parallel-axis
theorem by referring to Fig. a 10 mm

1 1
Ix = Σ ( Ix′ + Ad 2y ) ;  Ix = (140)(103) + 2 c (10)(1003) + 10(100)(452) d
12 12
= 5.7283 (106) mm4

1 1
Iy = Σ ( Iy′ + Ad 2x ) ;  Iy = (10)(1403) + 2 c (100)(103) + 100(10)(752) d
12 12
= 13.5533 (106) mm4

Ixy = Σ ( Ix′y′ + Ad x d y ) ;  Ixy = 0 + 30 + 10(100)( - 75)(45) 4

+ [0 + 10(100)(75)( - 45)] = - 6.75(106) mm4

Construction of the circle. The coordinates of center O of the circle are

Ix + Iy 5.7283 + 13.5533
a , 0b = a , 0b(106) = (9.6408, 0)(106)
2 2

And the reference point A is

(Ix, Ixy) = (5.7283, - 6.75)(106)


Thus, the radius of the circle is

R = OA = a 2(9.6408 - 5.7283)2 + ( - 6.75)2 b(106) = 7.8019(106)

Using these results, the circle shown in Fig. b can be constructed. Here, the coordinates of
points B and C represent Imin and Imax respectively. Thus

  Imax = (9.6408 + 7.8019)(106) = 17.44(106)mm4 = 17.4(106)mm4 Ans.

  Imin = (9.6408 - 7.8019)(106) = 1.8389(106)mm4 = 1.84(106)mm4 Ans.

1086
© 2016 Pearson Education, Inc., Upper Saddle River, NJ. All rights reserved. This material is protected under all copyright laws as they currently
exist. No portion of this material may be reproduced, in any form or by any means, without permission in writing from the publisher.

10–75. Continued

The orientation of the principal axes can be determined from the geometry of the
shaded triangle on the circle
6.75
  tan 2(up)2 =
9.6408 - 5.7283
2(up)2 = 59.90°

(up)2 = 29.95° = 30.0° b Ans.

And
2(up)1 = 180° - 2(up)1

2(up)1 = 180° - 59.90° = 120.10°

(up)1 = 60.04° = 60.0° d Ans.

Ans:
Imax = 17.4 ( 106 ) mm4
Imin = 1.84 ( 106 ) mm4
(up)2 = 30.0° b
(up)1 = 60.0° d

1087
© 2016 Pearson Education, Inc., Upper Saddle River, NJ. All rights reserved. This material is protected under all copyright laws as they currently
exist. No portion of this material may be reproduced, in any form or by any means, without permission in writing from the publisher.

*10–76.

Determine the orientation of the principal axes having an y


origin at point O, and the principal moments of inertia for
the rectangular area about these axes.
6 in.

3 in.

x
O
Solution
Moment And Product of Inertia About x and y Axes. Using the parallel-axis
theorem,
1
  Ix = Ix′ + Ad 2y;  Ix = (6)(33) + 6(3)(152) = 54.0 in4
12
1
  Iy = Iy′ + Ad 2x;  Iy = (3)(63) + 3(6)(32) = 216.0 in4
12
  Ixy = Ix′y′ + Ad xd y;  Ixy = 0 + 6(3)(3)(1.5) = 81.0 in4

Principal of Moment of Inertia.


Ix + Iy Ix - Iy 2
  I max = { a b + I 2xy
min 2 C 2

54.0 + 216.0 54.0 - 216.0 2


= { a b + 81.02
2 C 2

= 135 { 8122
Imax = 249.55 in4 = 250 in4 Ans.

Imin = 20.44 in4 = 20.4 in4 Ans.

The orientation of principal axes can now be determined.


- Ixy - 81.0
  tan 2up = = = 1.00
(Ix - Iy)>2 (54.0 - 216.0)>2
2up = 45.0°  And - 135°

up = 22.5°  And 67.5°

Substitute up = -67.5° into the equation for Iu,


Ix + Iy Ix + Iy
  Iu = + cos 2u - Ixy sin 2u
2 2
54.0 + 216.0 54.0 - 216.0
= + cos ( - 135°) - 81.0 sin ( -135°)
2 2
= 249.55 in4
thus, (up)1 = - 67.5°  (up)2 = 22.5° Ans.

Ans:
Imax = 250 in4
Imin = 20.4 in4
(up)1 = - 67.5°
(up)2 = 22.5°

1088
© 2016 Pearson Education, Inc., Upper Saddle River, NJ. All rights reserved. This material is protected under all copyright laws as they currently
exist. No portion of this material may be reproduced, in any form or by any means, without permission in writing from the publisher.

10–77.

Solve Prob. 10–76 using Mohr’s circle. y

6 in.

3 in.

x
O
Solution
Moment And Product of Inertia About x and y Axes. Using the parallel-axis
theorem,
1
  Ix = Ix′ + Ad 2y;  Ix = (6)(33) + 6(3)(1.52) = 54.0 in4
12
1
  Iy = Iy′ + Ad 2x;  Iy = (3)(63) + 3(6)(32) = 216.0 in4
12
  Ixy = Ix′y′ + Ad xd y;  Ixy = 0 + 6(3)(3)(1.5) = 81.0 in4

Construction of the circle. The coordinates of center o of the circle is


Ix + Iy 54.0 + 216.0
  a , 0b = a , 0b = (135, 0)
2 2
And the reference point A is

  (Ix, Ixy) = (54.0, 81.0)

Thus, the radius of the circle is

R = OA = 2(135 - 54.0)2 + 81.02 = 8122

using these results, the circle shown in Fig. a can be constructed. Here, the coordinated
of point B and C represent Imin and Imax respectively. Thus,

1089
© 2016 Pearson Education, Inc., Upper Saddle River, NJ. All rights reserved. This material is protected under all copyright laws as they currently
exist. No portion of this material may be reproduced, in any form or by any means, without permission in writing from the publisher.

10–77. Continued

  Imax = 135 + 8122 = 249.55 in4 = 250 in4 Ans.


4 4
  Imin = 135 - 8122 = 20.44 in = 20.4 in  Ans.

The orientation of the principal axes can be determined from the geometry of the
shaded triangle on the circle.

81.0
  tan 2(up)2 = = 1.00
135 - 54.0

2(up)2 = 45°
(up)2 = 22.5° d Ans.

And

2(up)1 = 180° - 2(up)2

2(up)1 = 180° - 45° = 135°


(up)1 = 67.5° b Ans.

Ans:
Imax = 250 in4
Imin = 20.4 in4
(up)2 = 22.5° d
(up)1 = 67.5° b

1090
© 2016 Pearson Education, Inc., Upper Saddle River, NJ. All rights reserved. This material is protected under all copyright laws as they currently
exist. No portion of this material may be reproduced, in any form or by any means, without permission in writing from the publisher.

10–78.

The area of the cross section of an airplane wing has the y


following properties about the x and y axes passing through
the centroid C: Ix = 450 in4, Iy = 1730 in4, Ixy = 138 in4.
Determine the orientation of the principal axes and the
principal moments of inertia.

x
C
SOLUTION
- 2Ixy - 2(138)
tan 2u = =
Ix - Iy 450 - 1730

u = 6.08° Ans.
Ix + Iy Ix - Iy 2
Imax/min = ; a b + I2xy
2 A 2

450 + 1730 450 - 1730 2


= ; a b + 1382
2 A 2

Imax = 1.74(103) in4 Ans.

Imin = 435 in4 Ans.

Ans:
u = 6.08°
Imax = 1.74 ( 103 ) in4
Imin = 435 in4

1091
© 2016 Pearson Education, Inc., Upper Saddle River, NJ. All rights reserved. This material is protected under all copyright laws as they currently
exist. No portion of this material may be reproduced, in any form or by any means, without permission in writing from the publisher.

10–79.

Solve Prob. 10–78 using Mohr’s circle. y

x
C
SOLUTION
- 2Ixy - 2(138)
tan 2u = =
I x - Iy 450 - 1730

u = 6.08° Ans.

Ix + Iy
Center of circle: 450 + 1730
= = 1090 in4
2 2
R = 2(1730 - 1090)2 + (138)2 = 654.71 in4

Imax = 1090 + 654.71 = 1.74(103) in4 Ans.

Imin = 1090 - 654.71 = 435 in4 Ans.

Ans:
u = 6.08°
Imax = 1.74 ( 103 ) in4
Imin = 435 in4

1092
© 2016 Pearson Education, Inc., Upper Saddle River, NJ. All rights reserved. This material is protected under all copyright laws as they currently
exist. No portion of this material may be reproduced, in any form or by any means, without permission in writing from the publisher.

*10–80.

Determine the moments and product of inertia for the y


u
shaded area with respect to the u and v axes. 10 mm

60
10 mm
x
120 mm 10 mm

Solution
Moment And Product of Inertia About x and y Axes. Since the x axis is an axis 120 mm
of symmetry, Ixy = 0. Also, the x axis passes through the centroids of the two 20 mm
segments, Fig. a
1 1
Ix = (20)(1203) + (120)(203) = 2.96(106) mm4
12 12
using the parallel-axis theorem,
1 1
Iy = (120)(203) + c (20)(1203) + 20(120)(702) d = 14.72(106) mm4
12 12
Moment And Product of Inertia About the Inclined u and v Axes. with u = 60°,
Ix + Iy Ix - Iy
Iu = + cos 2u - Ixy sin 2u
2 2
2.96 + 14.72 2.96 - 14.72
= a + cos 120° - 0 sin 120°b(106)
2 2
= 11.78(106) mm4 = 11.8(106) mm4

Ix + Iy Ix - Iy
Iv = - cos 2u + Ixy sin u
2 2
2.96 + 14.72 2.96 - 14.72
= a - cos 120° + 0 sin 120°b(106)
2 2
= 5.90(106) mm4

Ix - Iy
Iuv = sin 2u + Ixy cos 2u
2
2.96 - 14.72
= sin 120° + 0 cos 120°
2
= - 5.0922(106) mm4 = - 5.09(106) mm4

Ans:
Iu = 11.8(106) mm4
Iv = 5.90(106) mm4
Iuv = - 5.09(106) mm4

1093
© 2016 Pearson Education, Inc., Upper Saddle River, NJ. All rights reserved. This material is protected under all copyright laws as they currently
exist. No portion of this material may be reproduced, in any form or by any means, without permission in writing from the publisher.

10–81.

Solve Prob. 10–80 using Mohr’s circle. y


u
10 mm

60
10 mm
x
120 mm 10 mm

Solution
Moment And Product of Inertia About x and y Axes. Since the x axis is an axis 120 mm
of symmetry, Ixy = 0. Also, the x axis passes through the centroids of the two 20 mm
segments, Fig. a
1 1
Ix = (20)(1203) + (120)(203) = 2.96(106) mm4
12 12
using the parallel-axis theorem,
1 1
Iy = (120)(203) + c (20)(1203) + 20(120)(702) d = 14.72(106) mm4
12 12

Construction of the circle. The coordinate of center o of the circle is


Ix + Iy 2.96 + 14.72
a ,0b = a , 0b(106) = (8.84, 0)(106)
2 2
And the reference point A is
(Ix, Ixy) = (2.96, 0)(106)
Thus, the radius of the circle is

R = OA = a 2(8.84 - 2.96)2 + 02 b(106) = 5.88(106)

using these results, the circle shown in Fig. b can be constructed. Rotate radial
line OA counterclockwise 2u = 120° to coincide with radial line OP where the
coordinate of point P is ( Iu, Iuv ) . Then
Iu = (8.84 + 5.88 cos 60°)(106) = 11.78(106) mm4 = 11.8(106) mm4 Ans.

Iuv = 5.88(106) sin 60° = - 5.0922(106) mm4 = - 5.09(106) mm4 Ans.


Ir is represented by the coordinate of point Q. Thus
Iv = (8.84 - 5.88 cos 60°)(106) = 5.90(106) mm4 Ans.

Ans:
Iu = 11.8 ( 106 ) mm4
Iuv = - 5.09 ( 106 ) mm4
Iv = 5.90 ( 106 ) mm4

1094
© 2016 Pearson Education, Inc., Upper Saddle River, NJ. All rights reserved. This material is protected under all copyright laws as they currently
exist. No portion of this material may be reproduced, in any form or by any means, without permission in writing from the publisher.

10–82.

Determine the directions of the principal axes with origin y


located at point O, and the principal moments of inertia for
the area about these axes.
2 in. 2 in.

2 in.

1 in.
SOLUTION
4 in.
1 1
Ix = c (4)(6)3 + (4)(6)(3)2 d - c p(1)4 + p(1)2(4)2 d
12 4

= 236.95 in4 x
O
1 1
Iy = c (6)(4)3 + (4)(6)(2)2 d - c p(1)4 + p(1)2(2)2 d
12 4

= 114.65 in4

Ixy = [0 + (4)(6)(2)(3)] - [0 + p (1)(2)(4)] = 118.87 in4

-Ixy - 118.87
tan 2uP = =
Ix - Iy (236.95 - 114.65)
2 2

uP = -31.388°; 58.612°

Thus,

uP1 = - 31.4°; uP2 = 58.6° Ans.


Ix + Iy Ix - Iy 2
Imax = ; a b + I2xy
min 2 C 2

236.95 + 114.65 236.95 - 114.65 2


= ; a b + (118.87)2
2 C 2

Imax = 309 in4 Ans.

Imin = 42.1 in4 Ans.

Ans:
up1 = - 31.4°
up2 = 58.6°
Imax = 309 in4
Imin = 42.1 in4

1095
© 2016 Pearson Education, Inc., Upper Saddle River, NJ. All rights reserved. This material is protected under all copyright laws as they currently
exist. No portion of this material may be reproduced, in any form or by any means, without permission in writing from the publisher.

10–83.

Solve Prob. 10–82 using Mohr’s circle. y

2 in. 2 in.

2 in.

1 in.
SOLUTION
4 in.
1 1
Ix = c (4)(6)3 + (4)(6)(3)2 d - c p(1)4 + p(1)2(4)2 d
12 4

= 236.95 in4 x
O
1 1
Iy = c (6)(4)3 + (4)(6)(2)2 d - c p(1)4 + p(1)2(2)2 d
12 4

= 114.65 in4

Ixy = [0 + (4)(6)(2)(3)] - [0 + p (1)(2)(4)] = 118.87 in4

I x + Iy 236.95 + 114.65
Center of circle: = = 175.8 in4
2 2

R = 2(236.95 - 175.8)2 + (118.87)2 = 133.68 in4

Imax = (175.8 + 133.68) = 309 in4 Ans.

Imin = (175.8 - 133.68) = 42.1 in4 Ans.

118.87
2up1 = tan-1 a b = 62.78°
(236.95 - 175.8)

up1 = - 31.4° Ans.

up2 = 90° - 31.4° = 58.6° Ans.

Ans:
Imax = 309 in4
Imin = 42.1 in4
up1 = - 31.4°
up2 = 58.6°

1096
© 2016 Pearson Education, Inc., Upper Saddle River, NJ. All rights reserved. This material is protected under all copyright laws as they currently
exist. No portion of this material may be reproduced, in any form or by any means, without permission in writing from the publisher.

*10–84.

Determine the moment of inertia of the thin ring about the y


z axis. The ring has a mass m.

R
x

SOLUTION
2p
Iz = r A(R du)R2 = 2p r A R3
L0
2p
m = r A R du = 2p r A R
L0

Thus,

Iz = m R 2 Ans.

Ans:
Iz = m R2

1097
© 2016 Pearson Education, Inc., Upper Saddle River, NJ. All rights reserved. This material is protected under all copyright laws as they currently
exist. No portion of this material may be reproduced, in any form or by any means, without permission in writing from the publisher.

10–85.

Determine the moment of inertia of the ellipsoid with respect y


to the x axis and express the result in terms of the mass m of
the ellipsoid. The material has a constant density r. x2 y2
1
a2 b2

SOLUTION
a

dm= py2dx
L
y2dm
d Ix =
2
a 2
x 4
rp b a 1 - b dx =
2 2
m = r dV = 2
prab
LV L-a a 3
a
1 x2 2 8
Ix = rpb4 a 1 - 2 b dx = prab4
L-a 2 a 15

Thus,

2
Ix = mb2 Ans.
5

Ans:
2
Ix = mb2
5

1098
© 2016 Pearson Education, Inc., Upper Saddle River, NJ. All rights reserved. This material is protected under all copyright laws as they currently
exist. No portion of this material may be reproduced, in any form or by any means, without permission in writing from the publisher.

10–86.

Determine the radius of gyration kx of the paraboloid. The y


density of the material is r = 5 Mg>m3.
y2  50 x

100 mm

SOLUTION 200 mm

Differential Disk Element: The mass of the differential disk element is


dm = rdV = rpy2 dx = rp(50x) dx. The mass moment of inertia of this element
1 1 rp
is dIx = dmy2 = [rp(50x) dx](50x) = (2500x2) dx.
2 2 2
Total Mass: Performing the integration, we have
200 mm
m = dm = rp(50x)dx = rp(25x2)|200
0
mm
= 1(106)rp
Lm L0

Mass Moment of Inertia: Performing the integration, we have


200 mm
rp
Ix = dIx = (2500x2) dx
L L0 2

b`
rp 2500x3 200 mm
= a
2 3 0

= 3.333(109)rp

The radius of gyration is

Ix 3.333(109)rp
kx = = = 57.7 mm Ans.
Am A 1(106)rp

Ans:
kx = 57.7 mm

1099
© 2016 Pearson Education, Inc., Upper Saddle River, NJ. All rights reserved. This material is protected under all copyright laws as they currently
exist. No portion of this material may be reproduced, in any form or by any means, without permission in writing from the publisher.

10–87.

The paraboloid is formed by revolving the shaded area y


2
around the x axis. Determine the moment of inertia about y2 = a–h x
the x axis and express the result in terms of the total mass m
of the paraboloid. The material has a constant density r.
a

SOLUTION
h
2
dm = r dV = r (p y dx)

1 1
d Ix = dm y2 = r p y4 dx
2 2

h
1 a4
Ix = r p a 2 b x2 dx
L0 2 h
1
= p ra4 h
6

h
1 a2
m = r p a bx dx
L0 2 h
1
= r p a2 h
2
1
Ix = ma2 Ans.
3

Ans:
1
Ix = ma2
3

1100
© 2016 Pearson Education, Inc., Upper Saddle River, NJ. All rights reserved. This material is protected under all copyright laws as they currently
exist. No portion of this material may be reproduced, in any form or by any means, without permission in writing from the publisher.

*10–88.

Determine the moment of inertia of the homogenous z


triangular prism with respect to the y axis. Express the
result in terms of the mass m of the prism. Hint: For
integration, use thin plate elements parallel to the x-y plane z = –h
a (x – a)
having a thickness of dz.

h
SOLUTION
z
Differential Thin Plate Element: Here, x = a a 1 - b . The mass of the x
h b a
y
z
differential thin plate element is dm = rdV = rbxdz = rab a1 - b dz. The mass
h
moment of inertia of this element about y axis is

dIy = dIG + dmr2

dmx2 + dm ¢ + z2 ≤
1 x2
=
12 4

1 2
= x dm + z2 dm
3

= B a 1 - b + z2 R B raba 1 - b dz R
a2 z 2 z
3 h h

¢ a + 2 z2 -
3a2 3a2 a2 3z3
b dz
rab 2
= z - 3 z3 + 3z2 -
3 h h h h

Total Mass: Performing the integration, we have


h
b dz = rab ¢ z - ≤ ` = rabh
z z2 h 1
m = dm = raba 1 -
Lm L0 h 2h 0 2

Mass Moment of Inertia: Performing the integration, we have


h
¢ a + 2 z2 -
3a2 3a2 a2 3z3
b dz
rab 2
Iy = dIy = z - 3 z3 + 3z2 -
L L0 3 h h h h

¢ a z + 2 z3 - ≤`
rab 2 a2 3a2 2 a2 4 3 3z4 h
= z - z + z -
3 h 2h 4h3 4h 0

1a + h22
rabh 2
=
12

The mass moment of inertia expressed in terms of the total mass is

1 rabh m 2
Iy = a2 + h2 = a + h2 Ans.
6 2 6

Ans:
m 2
Iy = (a + h2)
6

1101
© 2016 Pearson Education, Inc., Upper Saddle River, NJ. All rights reserved. This material is protected under all copyright laws as they currently
exist. No portion of this material may be reproduced, in any form or by any means, without permission in writing from the publisher.

10–89.

Determine the moment of inertia of the semi-ellipsoid with y x2 y2


respect to the x axis and express the result in terms of the  1
a2 b2
mass m of the semiellipsoid. The material has a constant
density r.
b

SOLUTION
a

x2
Differential Disk Element: Here, y = b a 1 - b . The mass of the differential disk element is
2 2
a2
x2
dm = rdV = rp y2 dx = rp b2 a 1 - b dx. The mass moment of inertia of this element is
a2
1 1 x 2
x2 rp b4 x4 2x2
dIx = dmy2 = c rp b2 a 1 - 2 b dx d c b2 a 1 - 2 b d = a 4 - 2 + 1b dx.
2 2 a a 2 a a

Total Mass: Performing the integration, we have


a
b`
x2 x3 a
m = dm = rp b2 a 1 - b dx = rp b2
a x -
Lm L0 a2 3a2 0

2
= rpab2
3

Mass Moment of Inertia: Performing the integration, we have


a
rp b4 x4 2x2
Ix = dIx = a 4 - 2 + 1b dx
L L0 2 a a

rp b4 x5
`
2x3 a
= a 4 - + x b
2 5a 3a2 0

4
= rp ab4
15

The mass moment of inertia expressed in terms of the total mass is.

2 2 2
Ix = a rp ab2 b b2 = mb2 Ans.
5 3 5

Ans:
2
Ix = mb2
5

1102
© 2016 Pearson Education, Inc., Upper Saddle River, NJ. All rights reserved. This material is protected under all copyright laws as they currently
exist. No portion of this material may be reproduced, in any form or by any means, without permission in writing from the publisher.

10–90.
n
Determine the radius of gyration kx of the solid formed by y yn h x
revolving the shaded area about x axis. The density of the a
material is r.
h

Solution
Differential Disk Element. The mass of the differential disk element shown
h 1 h 1 2
shaded in Fig. a is dm = rdv = rpy2dx. Here y = 1 x n . Thus, dm = rp a 1 xn b
an an
rph2 2
dx = 2
xndx. The mass moment of Inertia of this element about the x axis is
an
1 1 rph2 n2 h 1 2 rxh4 4
dIx = (dm)y2 = a 2>n x dxb a 2 xn b = a xn dxb
2 2 a an
4
2an

Total Mass. Perform the integration,


a
rph2
Lm L0
2
m = dm = 2
(x ndx)
a n

rph2 n + 2
a
= a ba b axn n b 2
a
2
n n + 2 0

n
= a brpah2
n + 2
Mass Moment of Inertia. Perform the integration,
a
rph4 4
L L0 2an
Ix = dIx = 4
(xn dx)

a
rph4 n n+4
= a 4>n ba bax b3
2a n + 4 n
0

n
= c d rpah4
2(n + 4)
The radius of gyration is
n
c d rp ah4
Ix 2(n + 4) n + 2
kx = = = h Ans.
Am n A 2(n + 4)
a b rp ah2
c n+4

Ans:
n + 2
kx = h
A 2(n + 4)

1103
© 2016 Pearson Education, Inc., Upper Saddle River, NJ. All rights reserved. This material is protected under all copyright laws as they currently
exist. No portion of this material may be reproduced, in any form or by any means, without permission in writing from the publisher.

10–91.

The concrete shape is formed by rotating the shaded area y


about the y axis. Determine the moment of inertia Iy. The
specific weight of concrete is g = 150 lb>ft3.
4 in.
6 in.

2 2 8 in.
y x
9
SOLUTION x

1 1
d Iy = (dm)(10)2 - (dm)x2
2 2

1 1
= [pr (10)2 dy](10)2 - prx2 dyx2
2 2
8 8
pr B a b y2 dy R
1 9 2
Iy = (10)4 dy -
2 L0 L0 2
1
2 p (150)
B (10)4(8) - a b a b (8)3 R
2
9 1
= 3
32.2(12) 2 3

= 324.1 slug # in2

Iy = 2.25 slug # ft2 Ans.

Ans:
Iy = 2.25 slug # ft 2

1104
© 2016 Pearson Education, Inc., Upper Saddle River, NJ. All rights reserved. This material is protected under all copyright laws as they currently
exist. No portion of this material may be reproduced, in any form or by any means, without permission in writing from the publisher.

*10–92.

Determine the moment of inertia Ix of the sphere and


y
express the result in terms of the total mass m of the sphere. x2 y2 r2
The sphere has a constant density r.

r
x

SOLUTION
y2 dm
d Ix =
2

dm = r dV = r(py2 dx) = rp(r2 - x2)dx

1
d Ix = rp(r2 - x2)2 dx
2
r
1
Ix = rp(r2 - x2)2 dx
L-r 2

8
= prr5
15
r
m = rp(r2 - x2) dx
L-r

4
= rpr3
3

Thus,

2 2
Ix = mr Ans.
5

Ans:
2
Ix = mr 2
5

1105
© 2016 Pearson Education, Inc., Upper Saddle River, NJ. All rights reserved. This material is protected under all copyright laws as they currently
exist. No portion of this material may be reproduced, in any form or by any means, without permission in writing from the publisher.

10–93.

The right circular cone is formed by revolving the shaded y


area around the x axis. Determine the moment of inertia Ix
and express the result in terms of the total mass m of the y  –hr x
cone. The cone has a constant density r.
r

SOLUTION
dm = r dV = r(p y2 dx)
h
h
r(p) ¢ 2 ≤ x2 dx = rp ¢ 2 ≤ a b h3 = rp r2h
r2 r2 1 1
m =
L0 h h 3 3

1 2
dIx = y dm
2
1 2
= y (rp y2 dx)
2
1 r4
= r(p)a 4 b x4 dx
2 h

h
1 r4 1
Ix = r(p)a 4 b x4 dx = rp r4 h
L0 2 h 10
Thus,
3
Ix = m r2 Ans.
10

Ans:
3
Ix = mr 2
10

1106
© 2016 Pearson Education, Inc., Upper Saddle River, NJ. All rights reserved. This material is protected under all copyright laws as they currently
exist. No portion of this material may be reproduced, in any form or by any means, without permission in writing from the publisher.

10–94.

Determine the mass moment of inertia Iy of the solid z


formed by revolving the shaded area around the y axis. The
total mass of the solid is 1500 kg.
4m

z2 1 y3
––
16 2m

O y

SOLUTION x
Differential Element: The mass of the disk element shown shaded in Fig. a is
2
1 1
dm = rdV = rpr2dy. Here, r = z = y3>2.Thus, dm = rpa y3>2 b dy =
rp 3
y dy.
4 4 16
The mass moment of inertia of this element about the y axis is
dIy = dmr2 = A rpr2dy B r2 =
1 1 rp 1 3>2 4
a y b dy =
rp 4 rp 6
r dy = y dy.
2 2 2 2 4 512

Mass: The mass of the solid can be determined by integrating dm. Thus,
4m 4 4m
¢ ≤`
rp 3 rp y = 4 pr
m = dm = y dy =
L L0 16 16 4 0

The mass of the solid is m = 1500 kg. Thus,

375
1500 = 4pr r = kg>m3
p

Mass Moment of Inertia: Integrating dIy,


4m
rp y7 4 m
¢ ≤`
rp 6 32p
Iy = dIy = y dy = = r
L L0 512 512 7 0 7

375
Substituting r = kg>m3 into Iy,
p

b = 1.71(103) kg # m2
32p 375
Iy = a Ans.
7 p

Ans:
Iy = 1.71 ( 103 ) kg # m2

1107
© 2016 Pearson Education, Inc., Upper Saddle River, NJ. All rights reserved. This material is protected under all copyright laws as they currently
exist. No portion of this material may be reproduced, in any form or by any means, without permission in writing from the publisher.

10–95.

The slender rods have a mass of 4 kg>m. Determine the


moment of inertia of the assembly about an axis
perpendicular to the page and passing through point A. A

200 mm

Solution
100 mm 100 mm
Mass Moment of Inertia About An Axis Through A. The mass of each segment
is mi = (4 kg>m)(0.2 m) = 0.8 kg. The mass moment inertia of each segment
shown in Fig. a about an axis through their center of mass can be determined using
1
(IG)i = m l i2.
12 i

IA = Σ c (IG)i + mi d 2i d

1 1
= c (0.8) ( 0.22 ) + 0.8 ( 0.12 ) d + c (0.8) ( 0.22 ) + 0.8 ( 0.22 ) d
12 12

= 0.04533 kg # m2

= 0.0453 kg # m2  Ans.

Ans:
IA = 0.0453 kg # m2

1108
© 2016 Pearson Education, Inc., Upper Saddle River, NJ. All rights reserved. This material is protected under all copyright laws as they currently
exist. No portion of this material may be reproduced, in any form or by any means, without permission in writing from the publisher.

*10–96.

The pendulum consists of a 8-kg circular disk A, a 2-kg A


circular disk B, and a 4-kg slender rod. Determine the radius B
O
of gyration of the pendulum about an axis perpendicular to
the page and passing through point O.
1m 0.5 m
0.4 m 0.2 m

Solution
Mass Moment of Inertia About An Axis Through O. The mass moment of inertia
of each rod segment and disk segment shown in Fig. a about an axis passes through
1 1
their center of mass can be determined using (IG)i = m l i2 and (IG)i = mi r i2.
12 i 2

IO = Σ c (IG)i + mi d 2i d

1 1
   = c (4) ( 1.52 ) + 4 ( 0.252 ) d + c (2) ( 0.12 ) + 2 ( 0.62 ) d
12 2
1
   + c (8) ( 0.22 ) + 8 ( 1.22 ) d
2
   = 13.41 kg # m2
The total mass is
8 kg + 2 kg + 4 kg = 14 kg
The radius of gyration is
IO 13.41 kg # m2
kO = = = 0.9787 m = 0.979 m Ans.
Am C 14 kg

Ans:
kO = 0.979 m

1109
© 2016 Pearson Education, Inc., Upper Saddle River, NJ. All rights reserved. This material is protected under all copyright laws as they currently
exist. No portion of this material may be reproduced, in any form or by any means, without permission in writing from the publisher.

10–97.

Determine the moment of inertia Iz of the frustum of the z


0.2 m
cone which has a conical depression. The material has a
density of 200 kg>m3.

0.8 m
0.6 m

SOLUTION
3 1 0.4 m
Iz = [ p (0.4)2(1.6)(200)](0.4)2
10 3

3 1
- [ p (0.2)2(0.8)(200)](0.2)2
10 3

3 1
- [ p (0.4)2(0.6)(200)](0.4)2
10 3

Iz = 1.53 kg # m2 Ans.

Ans:
Iz = 1.53 kg # m2

1110
© 2016 Pearson Education, Inc., Upper Saddle River, NJ. All rights reserved. This material is protected under all copyright laws as they currently
exist. No portion of this material may be reproduced, in any form or by any means, without permission in writing from the publisher.

10–98.

The pendulum consists of the 3-kg slender rod and the 5-kg
thin plate. Determine the location y of the center of mass G
of the pendulum; then find the mass moment of inertia of O
the pendulum about an axis perpendicular to the page and
passing through G.
y
2m

SOLUTION G

©ym 1(3) + 2.25(5) 0.5 m


y = = = 1.781 m = 1.78 m Ans.
©m 3 + 5
1m
IG = ©IG¿ + md2

1 1
= (3)(2)2 + 3(1.781 - 1)2 + (5)(0.52 + 12) + 5(2.25 - 1.781)2
12 12

= 4.45 kg # m2 Ans.

Ans:
y = 1.78 m
IG = 4.45 kg # m2

1111
© 2016 Pearson Education, Inc., Upper Saddle River, NJ. All rights reserved. This material is protected under all copyright laws as they currently
exist. No portion of this material may be reproduced, in any form or by any means, without permission in writing from the publisher.

10–99.

Determine the mass moment of inertia of the thin plate


about an axis perpendicular to the page and passing O
through point O. The material has a mass per unit area of
20 kg>m2.
50 mm
150 mm

50 mm
150 mm
SOLUTION
400 mm 400 mm
Composite Parts: The plate can be subdivided into the segments shown in Fig. a.
Here, the four similar holes of which the perpendicular distances measured from
their centers of mass to point C are the same and can be grouped as segment (2).
This segment should be considered as a negative part. 150 mm 150 mm

Mass Moment of Inertia: The mass of segments (1) and (2) are m1 =
(0.4)(0.4)(20) = 3.2 kg and m2 = p(0.052)(20) = 0.05p kg, respectively. The mass
moment of inertia of the plate about an axis perpendicular to the page and passing
through point C is

1 1
IC = (3.2)(0.4 2 + 0.4 2) - 4 c (0.05p)(0.052) + 0.05p(0.152) d
12 2
= 0.07041 kg # m2

The mass moment of inertia of the wheel about an axis perpendicular to the
page and passing through point O can be determined using the parallel-axis
theorem IO = IC + md2, where m = m1 - m2 = 3.2 - 4(0.05p) = 2.5717 kg and
d = 0.4 sin 45°m. Thus,

IO = 0.07041 + 2.5717(0.4 sin 45°)2 = 0.276 kg # m2 Ans.

Ans:
IO = 0.276 kg # m2

1112
© 2016 Pearson Education, Inc., Upper Saddle River, NJ. All rights reserved. This material is protected under all copyright laws as they currently
exist. No portion of this material may be reproduced, in any form or by any means, without permission in writing from the publisher.

*10–100.

The pendulum consists of a plate having a weight of 12 lb O


and a slender rod having a weight of 4 lb. Determine the 1 ft
radius of gyration of the pendulum about an axis
perpendicular to the page and passing through point O. 1 ft
3 ft 2 ft

SOLUTION
IO = ©IG + md2

1 4 4 1 12 12
= a b (5)2 + a b (0.5)2 + a b (12 + 12) + a b(3.5)2
12 32.2 32.2 12 32.2 32.2

= 4.917 slug # ft2

4 12
m = a b + a b = 0.4969 slug
32.2 32.2

IO 4.917
kO = = = 3.15 ft Ans.
Am A 0.4969

Ans:
kO = 3.15 ft

1113
© 2016 Pearson Education, Inc., Upper Saddle River, NJ. All rights reserved. This material is protected under all copyright laws as they currently
exist. No portion of this material may be reproduced, in any form or by any means, without permission in writing from the publisher.

10–101.

If the large ring, small ring and each of the spokes weigh
100 lb, 15 lb, and 20 lb, respectively, determine the mass 4 ft
moment of inertia of the wheel about an axis perpendicular
to the page and passing through point A.
1 ft
O

SOLUTION
Composite Parts: The wheel can be subdivided into the segments shown in Fig. a. A
The spokes which have a length of (4 - 1) = 3 ft and a center of mass located at a
3
distance of a1 + b ft = 2.5 ft from point O can be grouped as segment (2).
2
Mass Moment of Inertia: First, we will compute the mass moment of inertia of the
wheel about an axis perpendicular to the page and passing through point O.

IO = ¢ ≤ (42) + 8 B ¢ ≤ (32) + ¢ ≤ (2.52) R + ¢ ≤ (12)


100 1 20 20 15
32.2 12 32.2 32.2 32.2

= 84.94 slug # ft2

The mass moment of inertia of the wheel about an axis perpendicular to the page and
passing through point A can be found using the parallel-axis theorem IA = IO + md2,

+ 8¢ ≤ +
100 20 15
where m = = 8.5404 slug and d = 4 ft. Thus,
32.2 32.2 32.2

IA = 84.94 + 8.5404(42) = 221.58 slug # ft2 = 222 slug # ft2 Ans.

Ans:
IA = 222 slug # ft 2

1114
© 2016 Pearson Education, Inc., Upper Saddle River, NJ. All rights reserved. This material is protected under all copyright laws as they currently
exist. No portion of this material may be reproduced, in any form or by any means, without permission in writing from the publisher.

10–102.

Determine the mass moment of inertia of the assembly z


about the z axis. The density of the material is 7.85 Mg> m3.

100 mm

SOLUTION
Composite Parts: The assembly can be subdivided into two circular cone segments (1)
and (3) and a hemispherical segment (2) as shown in Fig. a. Since segment (3) is a hole,
it should be considered as a negative part. From the similar triangles, we obtain 450 mm
z 0.1 300 mm
= z = 0.225m
0.45 + z 0.3
Mass: The mass of each segment is calculated as
300 mm
1 1
m1 = rV1 = r a pr2h b = 7.85(103) c p(0.32)(0.675) d = 158.9625p kg
3 3 x y

2 2
m2 = rV2 = r a pr3 b = 7.85(103) c p(0.33) d = 141.3p kg
3 3

1 1
m3 = rV3 = r a pr2h b = 7.85(103) c p(0.12)(0.225) d = 5.8875p kg
3 3

Mass Moment of Inertia: Since the z axis is parallel to the axis of the cone and the
hemisphere and passes through their center of mass, the mass moment of inertia can be
3 2 3
computed from (Iz)1 = m r12, (Iz)2 = m2r22, and m r32. Thus,
10 1 5 10 3
Iz = ©(Iz)i

3 2 3
= (158.9625p)(0.32) + (141.3p)(0.32) - (5.8875p)(0.12)
10 5 10

= 29.4 kg # m2 Ans.

Ans:
Iz = 29.4 kg # m2

1115
© 2016 Pearson Education, Inc., Upper Saddle River, NJ. All rights reserved. This material is protected under all copyright laws as they currently
exist. No portion of this material may be reproduced, in any form or by any means, without permission in writing from the publisher.

10–103.

Each of the three slender rods has a mass m. Determine the


moment of inertia of the assembly about an axis that is
perpendicular to the page and passes through the center
point O.
a a

SOLUTION

IO = 3 B ma2 + m ¢ ≤ R = ma2
1 a sin 60° 2 1 a
Ans.
12 3 2

Ans:
1
IO = ma2
2

1116
© 2016 Pearson Education, Inc., Upper Saddle River, NJ. All rights reserved. This material is protected under all copyright laws as they currently
exist. No portion of this material may be reproduced, in any form or by any means, without permission in writing from the publisher.

*10–104.

The thin plate has a mass per unit area of 10 kg>m2. z


Determine its mass moment of inertia about the y axis.
200 mm
200 mm
100 mm

200 mm

SOLUTION 100 mm

Composite Parts: The thin plate can be subdivided into segments as shown in Fig. a. 200 mm
Since the segments labeled (2) are both holes, the y should be considered as 200 mm
negative parts. 200 mm y
x 200 mm
200 mm
Mass moment of Inertia: The mass of segments (1) and (2) are
m1 = 0.4(0.4)(10) = 1.6 kg and m2 = p(0.12)(10) = 0.1p kg. The perpendicular
distances measured from the centroid of each segment to the y axis are indicated in
Fig. a. The mass moment of inertia of each segment about the y axis can be
determined using the parallel-axis theorem.

Iy = © A Iy B G + md2

1 1
= 2c (1.6)(0.42) + 1.6(0.22) d - 2 c (0.1p)(0.12) + 0.1p(0.22) d
12 4

= 0.144 kg # m2 Ans.

Ans:
Iy = 0.144 kg # m2

1117
© 2016 Pearson Education, Inc., Upper Saddle River, NJ. All rights reserved. This material is protected under all copyright laws as they currently
exist. No portion of this material may be reproduced, in any form or by any means, without permission in writing from the publisher.

10–105.

The thin plate has a mass per unit area of 10 kg>m2. z


Determine its mass moment of inertia about the z axis.
200 mm
200 mm
100 mm

200 mm

SOLUTION 100 mm

Composite Parts: The thin plate can be subdivided into four segments as shown in 200 mm
Fig. a. Since segments (3) and (4) are both holes, the y should be considered as 200 mm
negative parts. 200 mm y
x 200 mm
200 mm
Mass moment of Inertia: Here, the mass for segments (1), (2), (3), and (4) are
m1 = m2 = 0.4(0.4)(10) = 1.6 kg and m3 = m4 = p(0.12)(10) = 0.1p kg. The mass
moment of inertia of each segment about the z axis can be determined using the
parallel-axis theorem.

Iz = © A Iz B G + md2

1 1 1 1
= (1.6)(0.42) + c (1.6)(0.42 + 0.42) + 1.6(0.22) d - (0.1p)(0.12) - c (0.1p)(0.12) + 0.1p(0.22) d
12 12 4 2

= 0.113 kg # m2 Ans.

Ans:
Iz = 0.113 kg # m2

1118
© 2016 Pearson Education, Inc., Upper Saddle River, NJ. All rights reserved. This material is protected under all copyright laws as they currently
exist. No portion of this material may be reproduced, in any form or by any means, without permission in writing from the publisher.

10–106.

Determine the moment of inertia of the assembly about an


axis that is perpendicular to the page and passes through O
the center of mass G. The material has a specific weight of
g = 90 lb>ft3.
1 ft
G
2 ft
0.25 ft
SOLUTION
0.5 ft
1 90 1 90 1 ft
IG = ca b p(2.5)2(1) d (2.5)2 - c a b p(2)2(1) d(2)2
2 32.2 2 32.2
1 90 1 90
+ ca b p(2)2(0.25) d (2)2 - c a b p(1)2(0.25) d(1)2
2 32.2 2 32.2
= 118 slug # ft2 Ans.

Ans:
IG = 118 slug # ft 2

1119
© 2016 Pearson Education, Inc., Upper Saddle River, NJ. All rights reserved. This material is protected under all copyright laws as they currently
exist. No portion of this material may be reproduced, in any form or by any means, without permission in writing from the publisher.

10–107.

Determine the moment of inertia of the assembly about an


axis that is perpendicular to the page and passes through O
point O. The material has a specific weight of g = 90 lb>ft3.

1 ft
G
2 ft
0.25 ft
SOLUTION
0.5 ft
1 90 1 90 1 ft
IG = ca b p(2.5)2(1) d (2.5)2 - c a b p(2)2(1) d(2)2
2 32.2 2 32.2
1 90 1 90
+ ca b p(2)2(0.25) d (2)2 - c a b p(1)2(0.25) d(1)2
2 32.2 2 32.2
= 117.72 slug # ft2

IO = IG + md2

90 90
m = a b p(2 2 - 12)(0.25) + a b p(2.52 - 2 2)(1) = 26.343 slug
32.2 32.2

IO = 117.72 + 26.343(2.5)2 = 282 slug # ft2 Ans.

Ans:
IO = 282 slug # ft 2

1120
© 2016 Pearson Education, Inc., Upper Saddle River, NJ. All rights reserved. This material is protected under all copyright laws as they currently
exist. No portion of this material may be reproduced, in any form or by any means, without permission in writing from the publisher.

*10–108.

The pendulum consists of two slender rods AB and OC 0.4 m 0.4 m


which have a mass of 3 kg>m. The thin plate has a mass of A B
12 kg>m2. Determine the location y of the center of mass G
O
of the pendulum, then calculate the moment of inertia of
the pendulum about an axis perpendicular to the page and _
y
passing through G.
1.5 m
G
SOLUTION
1.5(3)(0.75) + p(0.3)2(12)(1.8) - p(0.1)2(12)(1.8)
y = C
1.5(3) + p(0.3)2(12) - p(0.1)2(12) + 0.8(3)
0.1 m
= 0.8878 m = 0.888 m Ans.
0.3 m
1
IG = (0.8)(3)(0.8)2 + 0.8(3)(0.8878)2
12

1
+ (1.5)(3)(1.5)2 + 1.5(3)(0.75 - 0.8878)2
12

1
+ [p(0.3)2(12)(0.3)2 + [p(0.3)2(12)](1.8 - 0.8878)2
2

1
- [p(0.1)2(12)(0.1)2 - [p(0.1)2(12)](1.8 - 0.8878)2
2

IG = 5.61 kg # m2 Ans.

Ans:
-
y = 0.888 m
IG = 5.61 kg # m2

1121
© 2016 Pearson Education, Inc., Upper Saddle River, NJ. All rights reserved. This material is protected under all copyright laws as they currently
exist. No portion of this material may be reproduced, in any form or by any means, without permission in writing from the publisher.

10–109.

Determine the moment of inertia Iz of the frustrum of the z


cone which has a conical depression. The material has a
density of 200 kg>m3.
200 mm

600 mm

SOLUTION 400 mm
z z + 1
Mass Moment of Inertia About z Axis: From similar triangles, = , 800 mm
0.2 0.8
z = 0.333 m. The mass moment of inertia of each cone about z axis can be
3
determined using Iz = mr2.
10

3 p
Iz = ©1Iz2i = c 10.82211.333212002 d10.822
10 3

3 p
- c 10.2 2210.333212002 d10.2 22
10 3

3 p
- c 10.2 2210.6212002 d10.2 22
10 3

= 34.2 kg # m2 Ans.

Ans:
Iz = 34.2 kg # m2

1122

You might also like